Download as pdf or txt
Download as pdf or txt
You are on page 1of 49

3

CHAPTER

MATRICES
Chapter Objectives
This chapter will help you understand :
¾¾ Matrices : Order of matrix; Types of matrices; Operations on matrices; Properties of multiplication of matrices; Transpose
of matrix; Symmetric and skew symmetric matrices; Elementary operations and Invertible matrix.

Quick Review
™™ Matrix is a concept of linear algebra, and it has wide
applications in many fields, including economics, TIPS…
statistics, computer programming, operations Setting up the Matrix for Solving.
research, industrial organisation and engineering.
™™ Multiplication of two matrices can be illustrated Write your equations in standard form.
from input-output models which are used to
illustrate producer theory in economics.
Transfer the numbers from the system of equations
™™ The concept of matrices originated in the work of into a matrix.
the two mathematicians Arthur Cayley and James Draw a large square bracket around full matrix.
Sylvester while solving a system of linear equations.
™™ In 1857, Cayley wrote Memoir on the Theory of
Matrices.
™™ Information content of a tree is given by the TRICKS…
negative log of the sum of the probabilities of the Recognise the form of the solution matrix.
trees it permits (Thorley et al., 1998).
Use scalar multiplication.
Know the Links Use row-addition or row-subtraction.
Combine row-addition and scalar multiplication in
☞☞
https://brilliant.org/wiki/matrices/ a single step.
☞☞
www.maths.surrey.ac.uk/explore/emmaspages/option1.
html Work from top to bottom first.
☞☞
https://www.britannica.com/science/matrix-mathematics

Multiple Choice Questions (1 mark each)

Q. 1. A =  aij  m×n is a square matrix, if −2


(c) y = 7, x =
3
(a) m < n (b) m > n −1 −2
(c) m = n (d) None of these (d) x = , y =
 [NCERT Ex. 3.1, Q. 8, Page 65]  3 3 [NCERT Ex. 3.1, Q. 9, Page 65]
Ans. Correct option : (c) Ans. Correct option : (b)
Explanation : It is known that a given matrix is said Explanation : It is given that
to be a square matrix if the number of rows is equal
to the number of columns. 3x + 7 5  0 y − 2 
Therefore,  y + 1 2 − 3x  = 8 4 
  
A = [ aif ]m×n is a square matrix, if m = n. Equating the corresponding elements, we get:
Q. 2. Which of the given values of x and y make the 3x + 7 = 0
following pair of matrices equal 7
⇒ x=−
3 x +7 5  0 y − 2  3
 y +1 2 − 3 x  , 8
   4  5= y−2

−1 ⇒ y=7
(a) x = , y = 7
3
(b) Not possible to find
MATRICES | 59
y +1 = 8 Q. 6. If A and B are symmetric matrices of same order,
AB – BA is a:
⇒ y=7
(a) Skew-symmetric matrix (b) Symmetric matrix
2 − 3x = 4
(c) Zero matrix (d) Identity matrix
2  [NCERT Ex. 3.3, Q. 11, Page 90]
⇒ x=−
3 Ans. Correct option : (a)
We find that on comparing the corresponding Explanation : Given that,
elements of the two matrices, we get two different A and B are symmetric matrices.
values of x, which is not possible. ⇒              A = A’ and B = B’
Hence, it is not possible to find the values of x and Now,      (AB – BA)’ = (AB)’ – (BA)’(i)
      

y for which the given matrices are equal. ⇒                  (AB – BA)’ = B’A’ – A’B’ [By reversal law]
Q. 3. The number of all possible matrices of order 3 × 3 ⇒                  (AB – BA)’ = BA – AB [From Eq. (i)]
with each entry 0 or 1 is: ⇒                  (AB – BA)’ = –(AB – BA)
(a) 27 (b) 18 ∴ (AB – BA) is a skew matrix.
(c) 81 (d) 512 cosa − sina 
 [NCERT Ex. 3.1, Q. 10, Page 65] Q. 7. If A =   , then A + A’ = I, then the
Ans. Correct option : (d)  sina cosa 
Explanation : The given matrix of the order 3 × 3 value of α is :
has 9 elements and each of these elements can be π π
(a) (b)
either 0 or 1. 6 3
Now, each of the 9 elements can be filled in two 3π
(c) π (d)
possible ways. 2
Therefore, by the multiplication principle, the  [NCERT Ex. 3.3, Q. 12, Page 90]
required number of possible matrices is 29 = 512. Ans. Correct option : (b)
Q. 4. Assume X, Y, Z, W and P are matrices of order 2 Explanation:
× n, 3 × k, 2 × p, n × 3 and p × k, respectively. The Given that,
restriction on n, k and p so that PY + WY will be cosa − sin a 
defined are: A=  
(a) k = 3, p = n (b) k is arbitrary, p = 2  sin a cosa 
Also A + A’ = I
(c) p is arbitrary, k = 3 (d) k = 2, p = 3
 [NCERT Ex. 3.2, Q. 21, Page 83] cosa − sin a   cosa sin a  1 0 
⇒ + =
Ans. Correct option : (a)  sin a cosa   − sin a cosa  0 1 
Explanation : Matrices P and Y are of the orders p ×  2 cosa 0  1 0 
k and 3 × k, respectively. Therefore, matrix PY will ⇒  0 =
 2 cosa  0 1 
be defined if k = 3. Consequently, PY will be of the
order p × k. Matrices W and Y are of the orders n × Equating corresponding entries, we have
3 and 3 × k respectively. ⇒ 2 cosa = 1
Since the number of columns in W is equal to the 1
number of rows in Y, matrix WY is well-defined ⇒ cosa =
2
and is of the order n × k. Matrices PY and WY can
p
be added only when their orders are the same. ⇒ cosa = cos
However, PY is of the order p × k and WY is of the 3
order n × k. Therefore, we must have p = n. Thus, k p
∴ a =
= 3 and p = n are the restrictions on n, k, and p so 3
that PY + WY will be defined. Q. 8. Matrices A and B will be inverse of each other
Q. 5. Assume X, Y, Z, W and P are matrices of order 2 × only if
n, 3 × k, 2 × p, n × 3 and p × k, respectively. If n = p, (a) AB = BA (b) AB = BA = 0
then the order of the matrix 7X – 5Z is: (c) AB = 0, BA = I (d) AB = BA = I
(a) p×2 (b) 2 × n  [NCERT Ex. 3.4, Q. 18, Page 97]
(c) n×3 (d) p × n Ans. Correct option : (d)
 [NCERT Ex. 3.2, Q. 22, Page 83] Explanation : We know that if A is a square matrix
Ans. Correct option : (b) of order m, and if there exists another square matrix
Explanation : Matrix X is of the order 2 × n. B of the same order m, such that AB = BA = I, then
Therefore, matrix 7X is also of the same order. B is said to be the inverse of A.
Matrix Z is of the order 2 × p, i.e., 2 × n [Since n = p] In this case, it is clear that A is the inverse of B.
Therefore, matrix 5Z is also of the same order. Thus, matrices A and B will be the inverse of each
Now, both the matrices 7X and 5Z are of the order other only if AB = BA = I.
2 × n. α β 
Thus, matrix 7X − 5Z is well-defined and is of the Q. 9. If A =   is such that A² = I, then
 γ −α 
order 2 × n.
(a) 1 + a² + bγ = 0
60 | OSWAAL NCERT SOLUTIONS – Textbook + Exemplar – MATHEMATICS : Class-XII

(b) 1 – a² + bγ = 0 ∴(I + A) − 7A = I
3

(c) 1 – a² – bγ = 0
(d) 1 + a² – bγ = 0 0 0 4 
 [NCERT Misc. Ex. Q. 13, Page 101] Q. 12. The matrix P =  0 4 0  is a
 
Ans. Correct option : (c)  4 0 0 
Explanation : (a) square matrix
a b  (b) diagonal matrix
A= 
 g −a  (c) unit matrix
∴ A2 = A ⋅ A (d) None of these
 [NCERT Exemp. Ex. 3.3, Q. 53, Page 59]
a b  a b 
Ans. Correct option : (a)
= ⋅
 g −a   g −a  Explanation : We know that, in a square matrix
a 2 + bg a b −ab  number of rows is equal to the number of columns.
=  0 0 4
a g − a g bg +a 2 
So, the matrix P =  0 4 0  is a square matrix.
a 2 + bg 0 
=   4 0 0 
 0 bg + a 2 
Q. 13. Total number of possible matrices of order 3 × 3
Now, with each entry 2 or 0 is
A2 = I (a) 9 (b) 27
a 2 + bg 0  1 0  (c) 81 (d) 512
 =   [NCERT Exemp. Ex. 3.3, Q. 54, Page 59]
 0 bg + a 2  0 1 
Ans. Correct option : (d)
On comparing the corresponding elements, we Explanation : Total number of possible matrices of
have : order 3 × 3 with each entry 2 or 0 is 29 i.e., 512.
a 2 + bg = 1  2 x + y 4 x   7 7 y − 13 
⇒ a 2 + bg − 1 = 0 Q. 14. If  5 x − 7 4 x  =  y x +6  , then the value of
   
⇒ 1 − a 2 − bg = 0 x + y is
Q. 10. If the matrix A is both symmetric and skew (a) x = 3, y= 1 (b) x = 2, y = 3
symmetric, then (c) x = 2, y= 4 (d) x = 3, y = 3
(a) A is a diagonal matrix  [NCERT Exemp. Ex. 3.3, Q. 55, Page 59]
(b) A is a zero matrix Ans. Correct option : (b)
(c) A is a square matrix Explanation : We have,
(d) None of these [NCERT Misc. Ex. Q. 14, Page 101] 4x = x + 6
Ans. Correct option : (b) ⇒ x=2
Explanation : If A is both symmetric and skew- And
symmetric matrices, then we should have, 4x = 7 y − 13
A ' = A and A ' = − A ⇒ 8 = 7 y − 13
⇒ A = −A
⇒ 7 y = 21
⇒ A+A=0
⇒ y=3
⇒ 2A = 0
∴x + y = 2 + 3 = 5
⇒ A=0
Therefore, A is a zero matrix.  −1 −1  x  
Q. 11. If A is square matrix such that A2 = A, then  sin ( xπ ) tan   
1  π 
(I + A)³ – 7A is equal to Q. 15. If A = and
π  −1  x  
(a) A (b) I – A  sin   cot (π x ) 
−1

(c) I (d) 3A  π  
 [NCERT Misc. Ex. Q. 15, Page 101]   x  
 −cos ( xπ ) tan   
−1 −1
Ans. Correct option : (c) 1 π
  
Explanation: B=  then A – B is
π −1  x 

(I + A)
3
− 7 A = I 3 + A 3 + 3I 2 A + 3 A 2 I − 7 A  sin π  − tan −1
( π x ) 
   
            = I + A + 3 A + 3 A − 7 A
3 2
equal to
  (a) I (b) O
              = I + A ⋅ A + 3 A + 3 A − 7 A Q A = A 
2 2

= I + A ⋅ A − A 1
              (c) 2I (d) I
             = I + A 2 − A 2
 [NCERT Exemp. Ex. 3.3, Q. 56, Page 60]
             = I + A − A
             = I Ans. Correct option : (d)
Explanation : We have,
MATRICES | 61

 −1        A = [aij]2 × 2, where aij= 1 if i ≠ j and 0 if i = j.


−1  x  
sin ( xp ) tan   
1  p  0 1
A= ∴A = 
p  −1  x   1 0 
 sin   cot (p x ) 
−1

 p
   And
and 0 1  0 1 
A2 =  ⋅
 −1  x   1 0  1 0 
 − cos ( xp ) tan   
−1

1 p   1 0 
B= = 
p −1  x 
 0 1 
 sin   − tan (p x ) 
−1

 p       =I
1 1 x x  1 0 0 
−1
(
 sin xp + cos xp
p
−1
) p 
tan −1 − tan −1  
p p  Q. 19. The matrix 0 2 0  is a
A−B= 
 1  −1 x x 1  0 0 4 
  sin − sin −1  ( )
cot −1 p x + tan −1 x 
p p p p  (a) identity matrix
(b) symmetric matrix
1  π    π 
−1 −1
(c) skew-symmetric matrix
  
π 2
0 Q sin x + cos x = 2 and 
=      (d) None of these
 π tan −1 x + cot −1 x = π   [NCERT Exemp. Ex. 3.3, Q. 60, Page 60]
 0   2 
 2 Ans. Correct option : (b)
1 1 0  1 0 0 
=  
2 0 1  Explanation : A = 0 2 0 
 
0 0 4 
1
= I
2 1 0 0 
 
Q. 16. If A and B are two matrices of the order 3 × m and ∴ A ' = 0 2 0  = A
3 × n, respectively, and m = n, then the order of 0 0 4 
matrix (5A – 2B) is
(a) m × 3 So, the given matrix is a symmetric matrix.
(b) 3 × 3 [Since, in a square matrix A, if A’ = A, then A is
(c) m × n called symmetric matrix.]
(d) 3 × n [NCERT Exemp. Ex. 3.3, Q. 57, Page 60] 0 −5 8 
Ans. Correct option : (d) Q. 20. The matrix  5 0 12  is a
Explanation : A3 × m and B3 × n are two matrices. If m 
 −8 −12 0 
= n, then A and B have same orders 3 × n as each,
so the order of (5A − 2B) should be same as 3 × n. (a) diagonal matrix
0 1  (b) symmetric matrix
2
Q. 17. If A =   then A is equal to (c) skew symmetric matrix
1 0 (d) scalar matrix
0 1 1 0   [NCERT Exemp. Ex. 3.3, Q. 61, Page 61]
(a) 
0 
(b)   Ans. Correct option : (c)
1 1 0 
Explanation : We know that, in a square matrix, if
0 1 1 0
(c)   (d)   bij, when i ≠ j then it is said to be a diagonal matrix.
0 1 0 1  Here, b12, b13…. ≠ 0 so the given matrix is not a
 [NCERT Exemp. Ex. 3.3, Q. 58, Page 60] diagonal matrix.
Ans. Correct option : (d)  0 −5 8 
Explanation: Now, B =  5 0 12 
A2 = A ⋅ A  −8 −12 0 
0 1  0 1 
 0 5 −8 
= ⋅ 
1 0  1 0  B ' =  −5 0 −12 
1 0  −8 12 0 
=
0 1 
 0 −5 8 
Q. 18. If matrix A = [aij]2 × 2, where aij= 1 if i ≠ j, and 0 if = −  5 0 12 

i = j then A2 is equal to
(a) I (b) A  −8 −12 0 
(c) 0 (d) None of these = −B
                      
 [NCERT Exemp. Ex. 3.3, Q. 59, Page 60] So, the given matrix is a skew-symmetric matrix,
Ans. Correct option : (a) since we know that in a square matrix B, if B’ = −B,
Explanation : We have, then it is called skew-symmetric matrix.
62 | OSWAAL NCERT SOLUTIONS – Textbook + Exemplar – MATHEMATICS : Class-XII

Q. 21. If A is matrix of order m × n and B is a matrix such Ans. Correct option : (d)
that AB′ and B′A are both defined, then order of Explanation : For any two matrices A and B, we
matrix B is may have AB = BA = I, AB ≠ BA and AB = 0 but it
(a) m × m (b) n × n is not always true.
(c) n × m (d) m × n Q. 25. On using elementary column operations C2 → C2
 [NCERT Exemp. Ex. 3.3, Q. 62, Page 61] − 2C1 in the following matrix equation
Ans. Correct option : (d)  1 −3   1 −1  3 1 
Explanation : Let, A =  aij  m×n and B = bij  p× q  2 4  = 0 1   2 4  we have
    
B ' = b ji 
q× p  1 −5  1 −1  3 −5
(a)  =  
Now, AB’ is defined, so n = q and B’A is also 0 4   −2 2   2 0 
defined, so p = m  1 −5  1 −1  3 −5
∴ Order of B ' = b ji 
(b)  =  
n× m 0 4  0 1   −0 2 
 1 −5  1 −3   3 1 
And order of B = bij  (c)  =  
 2 0  0 1   −2 4 
m× n
Q. 22. If A and B are matrices of same order, then (AB′ –
BA′) is a  1 −5  1 −1  3 −5
(d)  =  
(a) skew symmetric matrix  2 0  0 1   2 0 
(b) null matrix  [NCERT Exemp. Ex. 3.3, Q. 66, Page 61]
(c) symmetric matrix Ans. Correct option : (d)
(d) unit matrix Explanation : As we know that
 [NCERT Exemp. Ex. 3.3, Q. 63, Page 61] 1 −3 1 −1  3 1 
Ans. Correct option : (a)  2 4  = 0 1   2 4
    
Explanation : We have matrices A and B of same
order. By using C2 → C2 − 2C1 ,
Let, P ( AB '− BA ' )
1 −5 1 −1  3 −5
Then,  2 0  = 0 1   2 0 
P ' = ( AB '− BA ' ) '     
= ( AB ' ) '− ( BA ' ) ' Since, on using elementary column operation on X
= ( B ' ) ' ( A ) '− ( A ' ) ' B ' = AB, we apply these operations simultaneously
= BA '− AB ' on X and on the second matrix B of the product AB
on RHS.
= − ( AB '− BA ' )
Q. 26. On using elementary row operation R1 → R1 − 3R2
= −P
in the following matrix equation
Hence, ( AB '− BA ' ) is a skew symmetric matrix.
 4 2  1 2  2 0 
Q. 23. If A is a square matrix such that A2 = I, then (A −  3 3  = 0 3   1 1  we have
I)3 + (A + I)3 – 7A is equal to     
(a) A (b) I – A  −5 − 7   1 − 7   0
2
(a)  =
3  0 3   1 1 
(c) I+A (d) 3A 
3
 [NCERT Exemp. Ex. 3.3, Q. 64, Page 61]
 −5 −7   1 2   −1 −3 
Ans. Correct option : (a) (b)  =
Explanation : We have, A2 = I 3 3  0 3   1 1 
∴( A − I ) + ( A + I ) − 7A
3 3
 −5 −7  1 2   2 0 
(c)  =
3  1 −7   1 1 
 {2 2
}
= ( A − I ) + ( A + I ) ( A − I ) + ( A + I ) − ( A − I )( A + I ) 

3
 4 2   1 2  2 0 
 3 3
(
− 7 A Q a + b = ( a + b ) a + b − ab 
2 2
) (d)  =  
 −5 −7   −3 −3   1 1 
{ ( )}
= ( 2 A ) A + I − 2 AI + A 2 + I 2 + 2 AI − A 2 − I 2  − 7 A

2 2

 [NCERT Exemp. Ex. 3.3, Q. 67, Page 62]
Ans. Correct option : (a)
= 2 A  I + I + I + I − A + I  − 7 A Q A = AI 
2 2 2 2 2
 4 2  1 2   2 0 
= 2 A [ 5I − I ] − 7 A Explanation : We have,  =  
 3 3  0 3  1 1 
= 8 AI − 7 AI [Q A = AI ] Using elementary row operation, we have
= AI = A R1 → R1 − 3R2
Q. 24. For any two matrices A and B, we have  −5 −7  1 −7   2 0 
(a) AB = BA  =  
 3 3  0 3  1 1 
(b) AB ≠ BA
(c) AB = 0 Since, on using elementary row operation on X =
(d) None of the above AB, we apply these operations simultaneously on X
 [NCERT Exemp. Ex. 3.3, Q. 65, Page 61] and on the first matrix A of the product AB on RHS.
MATRICES | 63
Q. 27. _________ matrix is both symmetric and skew Q. 29. If A is skew-symmetric, then kA is a _________.
symmetric matrix. (where, k is any scalar).
 [NCERT Exemp. Ex. 3.3, Q. 68, Page 62]  [NCERT Exemp. Ex. 3.3, Q. 77, Page 63]
Ans. Null Ans. Skew-symmetric matrix.
Q. 28. The product of any matrix by the scalar _________ Q. 30. In applying one or more row operations while
is the null matrix. finding A–1 by elementary row operations, we
 [NCERT Exemp. Ex. 3.3, Q. 71, Page 62] obtain all zeroes in one or more, then A–1_________.
Ans. 0 (zero)  [NCERT Exemp. Ex. 3.3, Q. 81, Page 63]
Ans. Does not exist.

Very Short Answer Type Questions (1 or 2 marks each)

Q. 1. Sum of two skew symmetric matrices is always Q. 7. If A and B are square matrices of the same order,
_________ matrix. then
 [NCERT Exemp. Ex. 3.3, Q. 69, Page 62] (i) (AB)′ = _________.
Ans. Let A is a given matrix, then (−A) is a skew- (ii) (kA)′ = _________. (Where, k is any scalar)
symmetric matrix. Similarly, for a given matrix B is (iii) [k (A – B)]′ = _________.
a skew-symmetric matrix. [½]  [NCERT Exemp. Ex. 3.3, Q. 76, Page 63]
Hence, −A −B = –(A + B); sum of two skew- Ans. (i) (AB)’ = B’A’
symmetric matrices is always skew symmetric (ii) (kA)’ = kA’
matrix. [½] (iii) [k(A − B)]’ = k(A’ − B’) [2]
Q. 2. The negative of a matrix is obtained by multiplying Q. 8. If A and B are symmetric matrices, then
it by _________. (i) AB – BA is a _________.
 [NCERT Exemp. Ex. 3.3, Q. 70, Page 62] (ii) BA – 2AB is a _________.
Ans. Let A is a given matrix.  [NCERT Exemp. Ex. 3.3, Q. 78, Page 63]
∴ − A = −1[ A] Ans. (i) AB − BA is a skew-symmetric matrix.
So, the negative of a matrix is obtained by Since, [ AB − BA ] ' = ( AB ' ) − ( BA ) '
multiplying it by −1. [1]
= B ' A '− A ' B '     ( AB ) ' = B ' A ' 
Q. 3. A matrix which is not a square matrix is called a
_________ matrix. = BA − AB [ A ' = A and B ' = B]
 [NCERT Exemp. Ex. 3.3, Q. 72, Page 62] = − [ AB − BA ]
Ans. A matrix which is not a square matrix is called a
rectangular matrix. For example, a rectangular So, [AB − BA] is a skew-symmetric matrix. [1]
matrix is A=[aij ]m×n , where m ≠ n . [1] (ii) [BA − 2AB] is neither a symmetric, nor a skew-
symmetric matrix.
Q. 4. Matrix multiplication is _________ over addition.
∴ ( BA − 2 AB ) ' = ( BA ) '− 2 ( AB ) '
 [NCERT Exemp. Ex. 3.3, Q. 73, Page 62]
Ans. Matrix multiplication is distributive over addition. = A ' B ' − 2B ' A '
e.g., for three matrices A, B and C, we have = AB − 2BA
(i) A (B + C) = AB + AC = − ( 2BA − AB )
(ii) (A + B) C = AC + BC [2]
So, [BA − 2AB] is neither a symmetric, nor a skew-
Q. 5. If A is a symmetric matrix, then A3 is a _________
symmetric matrix. [1]
matrix. [NCERT Exemp. Ex. 3.3, Q. 74, Page 62]
Q. 9. If A is symmetric matrix, then B′AB is _________.
Ans. If A is a symmetric matrix, then A3 is a symmetric
matrix.  [NCERT Exemp. Ex. 3.3, Q. 79, Page 63]
 A' = A Ans. If A is a symmetric matrix, then B’AB is a symmetric
matrix. Then, B’AB is a symmetric matrix.
( )
∴ A 3 ' = A '3
 [B ' AB]' = B ' ( AB) '

n
( )
= A 3  ( A ' ) = A n ' 
 [1] = ( AB) ' (B ') '  ( AB ') ' = B ' A ' 
2
Q. 6. If A is a skew-symmetric matrix, then A is a = B' A'B
________. [NCERT Exemp. Ex. 3.3, Q. 75, Page 62] = [B ' A ' B ]
Ans. If A is a skew-symmetric matrix, then A2 is a
symmetric matrix. So, B’AB is a symmetric matrix. [2]
 A ' = −A Q. 10. If A and B are symmetric matrices of same order,
then AB is symmetric if and only if _________.
( )
∴ A 2 ' = A '2  [NCERT Exemp. Ex. 3.3, Q. 80, Page 63]
= (−A) [ A ' = − A ] Ans. If A and B are symmetric matrices of same order,
2

then AB is symmetric if and only if AB = BA.


= A2 [1]
64 | OSWAAL NCERT SOLUTIONS – Textbook + Exemplar – MATHEMATICS : Class-XII

Q. 20. State true or false in the given statement : If matrix


∴ ( AB ) ' = B ' A ' [Q AB = BA ] AB = 0, then A = 0 or B = 0 or both A and B are
= BA null matrices.
= AB [2]  [NCERT Exemp. Ex. 3.3, Q. 91, Page 63]
Q. 11. State true or false in the given statement : A matrix Ans. False, since for two non-zero matrices A and B of
denotes a number. same order, it can be possible that A.B = 0 = null
 [NCERT Exemp. Ex. 3.3, Q. 82, Page 63] matrix. [1]
Ans. False, a matrix is an ordered rectangular array of Q. 21. State true or false in the given statement :
numbers of functions. [1] Transpose of a column matrix is a column matrix.
Q. 12. State true or false in the given statement : Matrices  [NCERT Exemp. Ex. 3.3, Q. 92, Page 63]
of any order can be added. Ans. False, transpose of a column matrix is a row
 [NCERT Exemp. Ex. 3.3, Q. 83, Page 63] matrix. [1]
Ans. False, two matrices are added, if they are of the Q. 22. State true or false in the given statement : If A and
same order. [1] B are two square matrices of the same order, then
Q. 13. State true or false in the given statement : Two AB = BA. [NCERT Exemp. Ex. 3.3, Q. 93, Page 63]
matrices are equal if they have same number of Ans. False, for two square matrices of same order it is
rows and same number of columns. not always true that AB = BA. [1]
 [NCERT Exemp. Ex. 3.3, Q. 84, Page 63] Q. 23. State true or false in the given statement : If
Ans. False, if two matrices have same number of rows each of the three matrices of the same order are
and same number of columns, then they are said symmetric, then their sum is a symmetric matrix.
to be square matrix and if two square matrices have  [NCERT Exemp. Ex. 3.3, Q. 94, Page 63]
same elements in both the matrices, only then they Ans. True, let A, B and C are three matrices of same order.
are called equal. [2]     ∴ A ' = A , B ' = B and C ' = C
Q. 14. State true or false in the given statement : Matrices ∴ ( A + B + C ) ' = A '+ B '+ C '
of different order cannot be subtracted.
 [NCERT Exemp. Ex. 3.3, Q. 85, Page 63] = (A + B + C)
                                                 [2]
Ans. True, two matrices of same order can be Q. 24. State true or false in the given statement : If A and
subtracted. [1] B are any two matrices of the same order, then
Q. 15. State true or false in the given statement : Matrix (AB)′= A′B′.
addition is associative as well as commutative.  [NCERT Exemp. Ex. 3.3, Q. 95, Page 64]
 [NCERT Exemp. Ex. 3.3, Q. 86, Page 63] Ans. False, Q ( AB )' = B ' A ' [1]
Ans. True, matrix addition is associative as well as Q. 25. State true or false in the given statement : If (AB)′
commutative, i.e., = B′A′, where A and B are not square matrices,
( A + B) + C = A + ( B + C ) and A + B = B + A then number of rows in A is equal to number of
where A, B and C are matrices of same order. [2] columns in B and number of columns in A is equal
Q. 16. State true or false in the given statement : Matrix to number of rows in B.
multiplication is commutative.  [NCERT Exemp. Ex. 3.3, Q. 96, Page 64]
 [NCERT Exemp. Ex. 3.3, Q. 87, Page 63] Ans. True,
Ans. False, since it is possible when AB and BA are both Let, A is of order m × n and B is of order p × q
defined. [1] Since, (AB)’ = B’A’
Q. 17. State true or false in the given statement : A square ∴ A( m×n )Bp× q is defined.
matrix where every element is unity is called an
identity matrix. ⇒n=p
(i)
 [NCERT Exemp. Ex. 3.3, Q. 88, Page 63] And AB is of order m × q.
Ans. False, since AB ≠ BA in an identity matrix, the
⇒ ( AB ) ' is of order q × m.(ii)
diagonal elements are all one and rest are all zero. [1]
Also, B’ is of order q × p and A’ is of order n × m.
Q. 18. State true or false in the given statement : If A and
∴ B ' A ' is of order.
B are two square matrices of the same order, then ⇒p=n
A + B = B + A.
 [NCERT Exemp. Ex. 3.3, Q. 89, Page 63] And B’A’ is of order q × m.(iii)
Ans. True, since matrix addition is commutative, i.e., Also, equality of matrix (AB)’ = B’A’, we get the
A + B = B + A, where A and B are two square given statement as true.
matrices. [1] e.g., if A is order (3 × 1) and B is order of (1 × 3), we get
Q. 19. State true or false in the given statement : If A and Order of (AB)’ = Order of (B’A’) = 3 × 3 [2]
B are two matrices of the same order, then A – B = Q. 26. State true or false in the given statement : If A, B
B – A. [NCERT Exemp. Ex. 3.3, Q. 90, Page 63] and C are square matrices of same order, then AB
Ans. False, since the addition of two matrices of same = AC always implies that B = C.
order are commutative.  [NCERT Exemp. Ex. 3.3, Q. 97, Page 64]
∴ A + ( −B) = A − B = −[B − A] ≠ B − A Ans. False, if AB = AC = 0, then it can be possible that B
[2]
and C are two non-zero matrices such that B ≠ C.
MATRICES | 65
∴A ⋅B = 0 = A ⋅C [Q AB = BA ]
1 0 0 0  0 0 
Here, ( AB ) = ( AB )
−1 −1
Let, A =  , B=  and C =  3 1 
0 0   1 3   
1 0  0 0  ⇒ B −1 A −1 = A −1B −1
⇒                
∴ AB =  [Since A and B are satisfying commutative property
0 0  1 3
with respect to multiplications.] [2]
0 0  And  2 5 19 −7 
= 
0 0   
5
Q. 31. In the matrix A =  35 −2 12  write:
1 0  0 0   2 
        AC =   
0 0   3 1   3 1 −5 17 
0 0 (i) The order of the matrix,
=
0 0  (ii) The number of elements,
(iii) Write the elements a13, a21, a33, a24, a23.
          ⇒ AB = AC but B ≠ C [2]  [NCERT Ex. 3.1, Q. 1, Page 64]
Q. 27. State true or false in the given statement : AA′ is Ans. (i) In the given matrix, the number of rows is 3 and
always a symmetric matrix for any matrix A. the number of columns is 4. Therefore, the order of
 [NCERT Exemp. Ex. 3.3, Q. 98, Page 64] the matrix is 3 × 4. [1]
Ans. True, (ii) Since the order of the matrix is 3 × 4, there are
Q [ AA '] = ( A ')' A ' = [ AA '] 3 × 4 = 12 elements in it. [½]
[1]
5
2 3 (iii) a13 = 19, a21 = 35, a33 = –5, a24 = 12, a23 = [½]
2 3 −1 2
Q. 28. If A = and B = 4 5 then AB and BA Q. 32. If a matrix has 24 elements, what are the possible
1 4 2
2 1 orders it can have? What, if it has 13 elements?
are defined and equal.
 [NCERT Ex. 3.1, Q. 2, Page 64]
 [NCERT Exemp. Ex. 3.3, Q. 99, Page 64]
Ans. We know that if a matrix is of the order m × n, it has
Ans. False, since AB is defined, mn elements. Thus, to find all the possible orders of
 2 3 a matrix having 24 elements, we have to find all the
 2 3 −1  
∴ AB =    4 5 ordered pairs of natural numbers whose product is 24.
1 4 2   2 1 The ordered pairs are : (1, 24), (24, 1), (2, 12), (12, 2),
 
(3, 8), (8, 3), (4, 6) and (6, 4).
14 20 
=  Hence, the possible orders of a matrix having 24
 22 25 elements are:
Also, BA is defined. (1 × 24), (24 × 1), (2 × 12), (12 × 2), (3 × 8), (8 × 3), (4 ×
6) and (6 × 4)
 2 3
 2 3 −1 Thus (1, 13) and (13, 1) are the ordered pairs of
∴ BA =  4 5  
1 4 2 natural numbers whose product is 13.
 2 1  
Hence, the possible orders of a matrix having 13
 7 18 4  elements are 1 × 13 and 13 × 1. [2]
= 13 32 6  Q. 33. If a matrix has 18 elements, what are the possible
 5 10 0  orders it can have? What, if it has 5 elements?
 [NCERT Ex. 3.1, Q. 3, Page 64]
∴ AB ≠ BA [2] Ans. We know that if a matrix is of the order m × n, it has
Q. 29. State true or false in the given statement : If A is mn elements. Thus, to find all the possible orders of
skew-symmetric matrix, then A2 is a symmetric a matrix having 18 elements, we have to find all the
matrix. [NCERT Exemp. Ex. 3.3, Q. 100, Page 64] ordered pairs of natural numbers whose product is
Ans. True, 18. The ordered pairs are : (1, 18), (18, 1), (2, 9), (9,
Q  A 2  ' = [ A ']
2
2), (3, 6) and (6, 3).
Hence, the possible orders of a matrix having 18
= [− A] [Q A ' = − A ]
2
elements are:
=A 2
1 × 18, 18 × 1, 2 × 9, 9 × 2, 3 × 6, and 6 × 3
2
Hence, A is symmetric matrix. [2] Thus (1, 5) and (5, 1) are the ordered pairs of natural
Q. 30. (AB)–1 = A–1. B–1, where A and B are invertible numbers whose product is 5.
matrices satisfying commutative property with Hence, the possible orders of a matrix having 5
respect to multiplication. elements are 1 × 5 and 5 × 1. [2]
 [NCERT Exemp. Ex. 3.3, Q. 101, Page 64] Q. 34. Find the value of a, b, c and d from the equation:
Ans. True, we know that, if A and B are invertible  a − b 2 a + c   −1 5 
matrices of the same order, then  2 a − b 3 c + d  =  0 13 
   
( AB ) = ( BA )
−1 −1
 [NCERT Ex. 3.1, Q. 7, Page 64]
66 | OSWAAL NCERT SOLUTIONS – Textbook + Exemplar – MATHEMATICS : Class-XII

 a − b 2a + c   −1 5  11 11 0 
Ans.  =   
 2a − b 3c + d   0 13 = 16 5 21
As the two matrices are equal, their corresponding  5 10 9 
[2]
elements are also equal. (iv) cos2 x sin 2 x   sin 2 x cos2 x 
Comparing the corresponding elements, we get:  2 2 
+ 2 2 

a – b = –1 … (a11)  sin x cos x  cos x sin x 


From (a21), we have: cos2 x + sin 2 x sin 2 x + cos2 x 
= 2 2 
    b = 2a sin x + cos x cos x + sin x 
2 2

Then, from (a11), we have:


1 1
a – 2a = –1 =  Q sin 2 x + cos2 x = 1 [2]
⇒      a = 1 1 1   
⇒      b = 2 Q. 36. Compute the indicated products:
Now, from (a12), we have: (i)  a b   a − b 
2×1+c=5⇒c=3  − b a  b a 
From (a22) we have:   
3 × 3 + d = 13  
⇒   9 + d = 13 (ii)  2  [ 2 3 4 ]
 
⇒                  d = 4  
Therefore, a= 1, b = 2, c = 3 and d = 4. [2]
 1 −2   1 2 3 
Q. 35. Compute the following: (iii)   
 a b  a b  2 3   2 3 1
(i)  +  (iv)  2 3 4   1 −3 5 
 −b a  b a   
a2 + b2 b 2 + c 2   2 ab 2 bc   3 4 5  0 2 4 
(ii)  +   4 5 6   3 0 5 
2
a + c
2
a 2 + b 2   -2 ac −2 ab 
 2 1
 1 0 1
 −1 4 −6  12 7 6  (v)  3 2  
  −1 2 1
(iii)  8 5 16  +  8 0 5   −1 1   
   
 2 8 5   3 2 4 
 2 −3 
cos2 x sin 2 x   sin 2 x cos2 x   3 −1 3   
(vi)   1 0 
(iv)  sin 2 x cos2 x  + cos2 x sin 2 x   −1 0 2   3 1  [NCERT Ex. 3.2, Q. 3, Page 80]
      
 [NCERT Ex. 3.2, Q. 2, Page 80] Ans.
 a b   a −b 
Ans. (i)   
 −b a   b a 
(i)  a b  +  a b  =  a + a b + b 
       a (a) + b (b) a ( −b ) + b ( a ) 
 −b a   b a   −b + b a + a  = 
 −b ( a ) + a ( b ) −b ( −b ) + a ( a ) 
 2a 2b  [2]
=   a 2 + b 2 − ab + ab 
 0 2a  = 2 
 − ab + ab b + a 
2

(ii)  a 2 + b 2 b 2 + c 2   2ab 2bc 


 2 2
+ a2 + b2 0 
a + c
2
a +b  
2
−2 ac −2ab  = 
 0 a + b2 
2

 a 2 + b 2 + 2ab b 2 + c 2 + 2bc  [2]


= 2  1( 2 ) 1( 3) 1( 4 ) 
1 
 a + c − 2ac a + b − 2ab 
2 2 2

(ii) 2 [ 2 3 4] =  2 ( 2 ) 2 ( 3) 2 ( 4 ) 
 
( a + b ) 2 ( b + c ) 2   
=   3   3 ( 2 ) 3 ( 3) 3 ( 4 ) 
 ( a − c )2 ( a − b )2   
  [2] 2 3 4 
(iii)  −1 4 −6  12 7 6  =  4 6 8 
 8 5 16  +  8 0 5 
     6 9 12 
[2]
 2 8 5   3 2 4 
1 −2  1 2 3
 −1 + 12 4 + 7 −6 + 6  (iii)   
 2 3   2 3 1
=  8 + 8 5 + 0 16 + 5 
1(1) − 2 ( 2 ) 1( 2 ) − 2 ( 3) 1( 3) − 2 (1) 
 2 + 3 8 + 2 5 + 4  = 
 2 (1) + 3 ( 2 ) 2 ( 2 ) + 3 ( 3) 2 ( 3) + 3 (1) 
MATRICES | 67

1 − 4 2 − 6 3 − 2  Ans. 2 5 2 3 
= 3 1 1
 3 5 5
 2 + 6 4 + 9 6 + 3    
1 2 4 1 2 4
 −3 −4 1  3 A − 5B = 3  − 5
3 3 3 5 5 5
=     
 8 13 9  [2] 7 2 7 6 2
2
(iv)  2 3 4  1 −3 5   3 3   5 5 5 
 3 4 5  0 2 4  2 3 5 2 3 5
      
 4 5 6   3 0 5  = 1 2 4  − 1 2 4 
7 6 2  7 6 2 
 2 (1) + 3 ( 0 ) + 4 ( 3) 2 ( −3) + 3 ( 2 ) + 4 ( 0 )                               
 
3 (1) + 4 ( 0 ) + 5 ( 3) 3 ( −3) + 4 ( 2 ) + 5 ( 0 )  0 0 0 
 4 (1) + 5 ( 0 ) + 6 ( 3) 4 ( −3) + 5 ( 2 ) + 6 ( 0 )  = 0 0 0 
 
=  0 0 0 
                              [2]
2 5 + 3 4 + 4 5 
 ( ) ( ) ( )  Q. 38. Simplify
3 ( 5 ) + 4 ( 4 ) + 5 ( 5 )   cosq sinq   sinq −cosq 
  cosq  + sinq 
 4 ( 5 ) + 5 ( 4 ) + 6 ( 5 )   − sinq cosq  cosq sinq 

 2 + 0 + 12 −6 + 6 + 0 10 + 12 + 20   [NCERT Ex. 3.2, Q. 6, Page 81]



=  3 + 0 + 15 −9 + 8 + 0 15 + 16 + 25 
  cosq sin q   sin q − cosq 
Ans. cosq  + sin q 
cosq  
 4 + 0 + 18 −12 + 10 + 0 20 + 20 + 30   − sin q cosq sin q 
 cos2 q cosq sin q   sin 2 q − sin q cosq 
14 0 42  = + 
= 18 −1 56   − sin q cosq cos2 q  sin q cosq sin 2 q 

 22 −2 70   cos2 q + sin 2 q cosq sin q − sin q cosq 


[2] = 
 − sin q cosq + sin q cosq cos2 q + sin 2 q 
 2 1
 1 0 1  1 0 
(v)  3 2    =  (
Q cos2 q + sin 2 q = 1 [2] )
−1 2 1 0 1     
 −1 1  
 2 (1) + 1( −1) 2 ( 0 ) + 1( 2 ) 2 (1) + 1(1)  3 2   1 0
Q. 39. Find X, if Y =   and 2 X + Y =  .
  1 4   −3 2 
=  3 (1) + 2 ( −1) 3 ( 0 ) + 2 ( 2 ) 3 (1) + 2 (1) 
 −1(1) + 1( −1) −1( 0 ) + 1( 2 ) −1(1) + 1(1)   [NCERT Ex. 3.2, Q. 8, Page 81]
 
 2 −1 0 + 2 2 +1  Ans. 1 0
   1 2 3 2X + Y = 
=  3 − 2 0 + 4 3 + 2  =  1 4 5
 −3 2 
 −1 − 1 0 + 2 −1 + 1  −2 2 0 
   3 2  1 0
[2]
2X +  =
 2 −3 1 4   −3 2 
 3 −1 3   
(vi)   1 0   1 0  3 2 
 − 1 0 2  3 1  2X =  − 
   −3 2  1 4 
 3 ( 2 ) − 1(1) + 3 ( 3) 3 ( −3) − 1( 0 ) + 3 (1)   1 − 3 0 − 2
=  = 
 −1( 2 ) + 0 (1 ) + 2 ( 3 ) − 1( −3) + 0 ( 0 ) + 2 (1)   −3 − 1 2 − 4 
 6 − 1 + 9 −9 − 0 + 3  −2 −2 
=  2X =  
 −2 + 0 + 6 3 + 0 + 2   −4 −2 
14 −6  1  −2 −2 
=  X=  
4 5  2  −4 −2 
[2]
2 5 2 3   −1 −1
1 1 = 
3 3 5 5  −2 −1
    [2]
1 2 4 1 2 4
Q. 37. If A =  and B =  then Q. 40. Find the value of x and y, if
3 3 3 5 5 5
     1 3   y 0  5 6 
7 2
2 7 6 2 2 + = .
0 x   1 2  1 8 
 3 3   5 5 5 
compute 3A – 5B. [NCERT Ex. 3.2, Q. 5, Page 81]  [NCERT Ex. 3.2, Q. 9, Page 81]
   
 3 2  1 2
 −2 1   −2 0 
= + 
 3 2  2 4
68 |  −2 − 2 1 + 0 
OSWAAL NCERT SOLUTIONS – Textbook + Exemplar – MATHEMATICS : Class-XII
= 
 3 + 2 2 + 4
1 3  y 0  5 6   −4 1 
Ans. 2 + = = 
0 x  1 2  1 8   5 6
 2 6  y 0  5 6  Therefore,
⇒  + =
0 2x   1 2  1 8  '
 −4 1 
2 + y 6  5 6  ( A + 2B ) ' =  5 6

⇒  = 
 1 2 x + 2  1 8 
[1]  −4 5 
= 
Comparing the corresponding elements of these  1 6 [2]
two matrices, we have Q. 43. For the matrices A and B, verify that (AB)’ = B’A’,
2+ y =5 where:
⇒ y=3 1
2x + 2 = 8 (i) A =  −4  , B = [ −1 2 1]
⇒ x=3  3 
Therefore, x = 3 and y = 3. [1]
0 
Q. 41. Find the transpose of each of the following
matrices: (ii) A =  1  , B = [ 1 5 7 ]
5  2 
 [NCERT Ex. 3.3, Q. 5, Page 88]
   1 −1 Ans.
1
(i)   (ii)  
2 2 3  1
 −1  
  (i)        AB =  −4  [ −1 2 1]
 −1 5 6  3 
 
(iii)  3 5 6  −1 2 1 
  2 3 −1 [NCERT Ex. 3.3, Q. 1, Page 88] =  4 −8 −4 

 −3 6 3 
Ans.
5 LHS = ( AB ) '
1
'
(i) Let, A =    −1 2 1 
2
 −1 =  4 −8 −4 
   −3 6 3 
 1 
Transpose of A = A′ or AT = 5 −1 [2]  −1 4 −3
 2 
1 −1 =  2 −8 6 
(ii)  
2 3   1 −4 3 
1 2
Transpose of A = A′ or AT =  [2] RHS = B ' A '
 −1 3 
 −1 5 6  1
'
  ' 
(iii)  3 5 6  = −1 2 1  −4 
 2 3 −1  −1  3 
  3 2
 
Transpose of A = A′ or AT =  5 5 3 [2]  −1
6 6 −1  
 =  2  1 −4 3
 1 
 −2 3   −1 0 
Q. 42. If A' =   and B =   then find (A + 2B)’.  −1 4 −3
 1 2  1 2  
 [NCERT Ex. 3.3, Q. 4, Page 88] =  2 −8 6 
Ans. Given that,  1 −4 3 
 −2 3   −1 0   −2 1  ∴ LHS=RHS
A' =   and B =  1 2 , then ( A ' ) ' = A =  3 2 
 1 2      Hence proved. [2]
 −2 1  −1 0  0
A + 2B =  + 2  
3 2
 1 2
 (ii) AB = 1  [1 5 7 ]
 2 
 −2 1   −2 0 
= + 
3 2  2 4 0 0 0 
 −2 − 2 1 + 0  = 1 5 7 
=   2 10 14 
 3 + 2 2 + 4
 −4 1 
= 
 5 6
MATRICES | 69

LHS ( AB ) '  1 −1 5 
Q. 45. (i) Show that the matrix A =  −1 2 1 is a
0 0 0  
1  5 1 3 
 5 7 
 2 10 14  symmetric matrix.

0 1 2  0 1 −1
0  
 5 10  (ii) Show that the matrix A =  −1 0 1  is a skew
0 7 14  symmetric matrix.  1 −1 0 
RHS = B ' A '  [NCERT Ex. 3.3, Q. 7, Page 89]
Ans. (i) Given that,
0 '
= [1 5 7 ] 1 
'  1 −1 5
 
 2  A =  −1 2 1  …(i)
 5 1 3
1 
=  5  [ 0 1 2] Changing rows of matrix, A as the columns of new
7   1 −1 5
matrix A ' =  −1 2 1  = A
0 1 2 
 5 1 3
= 0 5 10 
0 7 14  ∴ A' = A
Therefore, by definitions of symmetric matrix, A is
∴ LHS=RHS a symmetric matrix. [2]
Hence proved. [2] (ii) Given that,
0 1 1
 cosα sinα 
Q. 44. (i) If A =   then verify that A’A = I. =  −1 0 1  …(i)
 − sinα cosα   1 1 0 
 sinα cosα 
(ii) If A =   then verify that A’A = I.  0 1 −1
'

 −cosα sinα 
A ' =  −1 0 1 
 [NCERT Ex. 3.3, Q. 6, Page 89]
Ans.  1 −1 0 
LHS = A ' A  0 −1 1 
 cos α sin α  '  cos α sin α  =  1 0 −1
(i) =   
 − sin α cos α   − sin α cos α   −1 1 0 
cos α − sin α   cos α sin α  −1
=   0 1
 sin α cos α   − sin α cos α  = −
 
Taking (−1) common, A '  −1 0 1  = −A
 cos2 α + sin 2 α cos α sin α − sin α cos α 
= 
[From Eq. (i)]  1 −1 0 
sin α cos α − cos α sin α sin 2 α + cos2 α  Therefore, by definition matrix A is a skew-
1 0  symmetric matrix [2]
= 
0 1  1 5
Q. 46. For a matrix A =   verify that:
   =I=RHS [2] 6 7 
(i) (A + A’) is a symmetric matrix.
(ii) LHS=A ' A
1 (ii) (A – A’) is a skew symmetric matrix.
 sin a cosa   sin a cosa 
=     [NCERT Ex. 3.3, Q. 8, Page 89]
 − cosa sin a   − cosa sin a  Ans. (i) Given that,
 sin a − cosa   sin a cosa  1 5 
=    A= 
cosa sin a   − cosa sin a 
6 7 
 sin 2 a + cos2 a sin a cosa − cosa sin a  Let,
=  B = A + A'
cosa sin a − sin a cosa cos2 a + sin 2 a 
'
1 5  1 5 
1 0  = +
=  6 7  6 7 
0 1 
1 5  1 6 
=I=RHS [2] = + 
6 7  5 7 
70 | OSWAAL NCERT SOLUTIONS – Textbook + Exemplar – MATHEMATICS : Class-XII

 1 + 1 5 + 6 Q. 48. Find the inverse of each of the matrices, if it exists


=  in given matrices.
6 + 5 7 + 7 
 2 1
 2 11  
=   1 1  [NCERT Ex. 3.4, Q. 2, Page 97]
11 14 
Ans.
 2 11 '  2 11  2 1
∴B' =   = =B Let, A =  
11 14  11 14  1 1
∴ B = A + A ' is a symmetric matrix. [2] We know that A = IA
(ii) Given that,  2 1 1 0 
1 5  ∴  = A
A=  1 1 1 1 
6 7  1 0  1 −1
Let, ⇒  =  A ( R1 → R1 − R2 )
1 1  0 1 
B = A − A'
1 0   1 −1
1 5  1 5  ' ⇒  =  A ( R2 → R2 − R1 )
= − 0 1   −1 2 
6 7  6 7 
 1 −1
1 5  1 6  ∴ A −1 =  
= −   −1 2  [2]
6 7  5 7 
Q. 49. Find the inverse of each of the matrices, if it exists
 1 − 1 5 − 6 in given matrices.
= 
6 − 5 7 − 7  1 3
0 −1 2 7 
=    [NCERT Ex. 3.4, Q. 3, Page 97]
1 0 
1 3 
A= 
0 −1 '  0 1  2 7
∴B' =   =  Ans.
1 0   −1 0  We know that A = IA
0 −1  1 3  1 0 
Taking ( −1) common, −   = −B ∴  = A
1 0   2 7  0 1 
   ∴ B = A − A ' is a skew-symmetric matrix. [2] 1 3  1 0 
⇒  = A ( R2 → R2 − 2R1 )
Q. 47. Using elementary transformations, find the 0 1  −2 1 
inverse of each of the matrices, if it exists in given 1 0   7 −3
 1 −1 ⇒  = A ( R1 → R1 − 3R2 )
matrices.  . 0 1   −2 1 
  2 3  [NCERT Ex. 3.4, Q. 1, Page 97]  7 −3
Ans. ∴ A −1 =  
 −2 1  [2]
1 −1
Let, A =   Q. 50. Find the inverse of each of the matrices, if it exists
2 3  in given matritces.
We know that A = IA 2 1
1 −1 1 0  4 2
∴ = A   [NCERT Ex. 3.4, Q. 14, Page 97]
 2 3  0 1  Ans.
1 −1  1 0  2 1
⇒ = A ( R2 → R2 − 2R1 ) Let, A =  
0 5   −2 1  4 2
 1 0 We know that A = IA
1 −1    1 
⇒  = 2 1 A  R2 → 5 R2   2 1  1 0 
0 1   −   ∴ = A
 5 5   4 2  0 1 
 3 1 1
Applying R1 → R1 − R2 , we have
1 0   2 5 2
⇒ =  ( R1 → R1 + R2 )
0 1   − 2 1  1 
 0 0  1 − 
 5 5  4 2 =  2 A
  0 1 
 3 1  
 5 Now, in the above equation, we can see that all the
∴A =  5
−1

− 2 1 zeros in the first row of the matrix is on the LHS.
 5 5  Therefore, A−1 does not exist. [2]
[2]
MATRICES | 71
Q. 51. If A and B are symmetric matrices, prove that AB (iii) elements a23, a31 and a12
− BA is a skew symmetric matrix.  [NCERT Exemp. Ex. 3.3, Q. 2, Page 52]
 [NCERT Misc. Ex. Q. 4, Page 100]  
Ans. It is given that A and B are symmetric matrices. a 1 x 
Therefore, we have  
Ans. We have, A =  2 3 x2 − y
A ' = A and B ' = B …(i)  −2 
0 5 
Now, ( AB − BA ) ' = ( AB ) '− ( BA ) ' ( A − B ) ' = A '− B '   5 
(i) The order of matrix A = 3 × 3 [½]
= B ' A '− A ' B ' ( AB ) ' = B ' A ' 
(ii) The number of elements = 3 × 3 = 9
= BA − AB  Using Eq. ( i )  [Since, the number of elements in an m x n matrix
will be equal to m × n = mn] [½]
= − ( AB − BA ) (iii) a23 = x2 − y, a31 = 0, a12 = 1
∴ ( AB − BA ) ' = − ( AB − BA ) [Since, we know that aij is a representation of
element tying in the ith row and jth column.] [1]
Thus, (AB – BA) is a skew-symmetric matrix. [2] Q. 55. Construct a2×2 matrix where:
1 2 0 0  (i − 2 j)
2

   (i) aij =
Q. 52. For what values of x : [ 1 2 1]  2 0 1   2  = 0 2
 1 0 2   x  (ii) aij = −2i +3 j
 [NCERT Misc. Ex. Q. 7, Page 100]  [NCERT Exemp. Ex. 3.3, Q. 3, Page 53]
Ans. We have, Ans. We know that, the notation A[aij]m × n, namely it
1 2 0  0  indicates that A is a matrix of order m × n.
[1 2 1] 2 0 1  2  = 0 (i) Here, A =  aij 
2× 2
 1 0 2   x 
(i − 2 j )
2

0 ⇒ A= , 1 ≤ i ≤ 2; 1 ≤ j ≤ 2
2
⇒ [1 + 4 + 1 2 + 0 + 0 0 + 2 + 2]  2  = 0
(1 − 2 )
2
1
 x  ∴ a11 = =
2 2
0 (1 − 2 × 2 )
2
9
⇒ [6 2 4]  2  = 0 a12 =
2
=
2
 x 
( 2 − 2 × 1)
2

a21 = =0
⇒ 6 ( 0 ) + 2 ( 2 ) + 4 ( x )  = 0 2
[ 4 + 4 x ] = [ 0] ( 2 − 2 × 2)
2

a22 = =2 …(i)
⇒ 4 + 4x = 0 2
Thus, the required value of x is –1. [2] 1 9
Q. 53. If a matrix has 28 elements, what are the possible Thus, A =  2 2 [2]
orders it can have? What if it has 13 elements?  
 0 2  2× 2
 [NCERT Exemp. Ex. 3.3, Q. 1, Page 52]
Ans. We know that, if a matrix is of order m × n it (ii) Here, A =  aij  = −2i + 3 j , 1 ≤ i ≤ 2 ; 1 ≤ j ≤ 2
2× 2
has mn elements, where m and n are natural
numbers. We have, ∴ a11 = −2 × 1 + 3 × 1 = 1
m × n = 28 a12 = −2 × 1 + 3 × 2 = 4 Q −1 = 1
⇒ ( m, n ) = {(1, 28 ) , ( 2,14 ) , ( 4, 7 ) , ( 7 , 4 ) , (14, 2 ) , ( 28,1)}
a21 = −2 × 2 + 3 × 1 = 1
So, the possible orders are : a22 = −2 × 2 + 3 × 2 = 2
1 × 28, 2 × 14, 4 × 7 , 7 × 4, 14 × 2, 28 × 1
1 4
Also, if it has 13 elements, then m × n = 13 ∴ A=
1 2 2× 2
( m, n ) = {(1,13) , (13,1)} [2]
Hence, the possible orders are 1 × 13, 13 × 1 [2] Q. 56. Construct a matrix 3 × 2 whose elements are given
  by aij = e ix sinjx .
a 1 x   [NCERT Exemp. Ex. 3.3, Q. 4, Page 53]
 
Q. 54. In the matrix A =  2 3 x 2 − y  write: Ans. Since, A =  aij 
 1 ≤ i ≤ m and 1 ≤ j ≤ n, i , j ∈ N
 −2 
m× n

0 5  ∴ A =  e i ⋅x sin jx  ; 1 ≤ i ≤ 3; 1 ≤ j ≤ 2
 5        3× 2

(i) the order of the matrix A ⇒ a11 = e1⋅x ⋅ sin 1 ⋅ x = e x sin x


(ii) the number of elements
a12 = e1⋅x ⋅ sin 2 ⋅ x = e x sin 2x
         
72 | OSWAAL NCERT SOLUTIONS – Textbook + Exemplar – MATHEMATICS : Class-XII

a21 = e 2⋅x ⋅ sin 1 ⋅ x = e 2 x sin x  5 2 −2 


          = 
a22 = e 2⋅x ⋅ sin 2 ⋅ x = e 2 x sin 2x 12 0 1 
         
3 1 −1
a31 = e 3⋅x ⋅ sin 1 ⋅ x = e 3x sin x (ii) 2X = 2  
          5 −2 −3 [2]
a32 = e 3⋅x ⋅ sin 2 ⋅ x = e 3x sin 2x  6 2 −2 
          = 
 e x sin x e x sin 2x  10 −4 −6 
 
A =  e 2 x sin x e 2 x sin 2x 
2 1 −1
 e sin x e sin 2x 
x x and 3Y = 3 
2 4 
3 3
  3× 2
[2] 7
Q. 57. Find values of a and b if A = B, where 6 3 −3
=
 a + 4 3b   2 a + 2 b2 + 2   21 6 12 
A=  and B =  .
 8 −6   8 b 2 − 5b   6−6 2 − 3 −2 + 3 
2 X − 3Y =  
 [NCERT Exemp. Ex. 3.3, Q. 5, Page 53] 10 − 21 −4 − 6 −6 − 12 
Ans. By equality of matrices we know that each element
of A is equal to the corresponding element of B,  0 −1 1  [2]
= 
that is aij = bij for all i and j.  −11 −10 −18
∴ a11 = b11
3 + 2 1 + 1 −1 − 1 
⇒ a + 4 = 2a + 2 (iii) X + Y = 
5 + 7 −2 + 2 −3 + 4 
⇒ a=2
5 2 −2 
a12 = b12 =
12 0 +1
⇒ 3b = b 2 + 2
⇒ b 2 = 3b − 2 0 0 0 
Also, X + Y + Z =  
and a22 = b22 0 0 0 
⇒ −6 = b 2 − 5b We see that Z is the additive inverse of (X+Y )
⇒ −6 = 3b − 2 − 5b Q b 2 = 3b − 2  or negative of (X+Y ).
⇒ 2b = 4  −5 −2 2 
Z=  Q Z = − ( X + Y )  [2]
⇒ b=2  −12 0 −1
∴ a = 2 and b = 2 [2] Q. 60. Find non-zero values of a : satisfying the matrix
Q. 58. If possible, find the sum of the matrices A and B, 2 x 2  8 5 x  ( + 8) 24 
 3 1 equation   + 2  = 2 .
x y z
where A =   and B =  .  3 x 4 4 x   (10) 6 
 2 3  a b 6  [NCERT Exemp. Ex. 3.3, Q. 8, Page 53]
 [NCERT Exemp. Ex. 3.3, Q. 6, Page 53] Ans. Given that,
Ans. We have,  2x 2   8 5x  ( x 2 + 8) 24 
 3 1 x  + 2  = 2 
x y z   3 x  4 4x 
A=  and B =    (10) 6x 
 2 3 2× 2  a b 6  2× 3  2x 2 2x  16 10x   2x 2 + 16 48 
Here, A and B are of different orders. Also, we ⇒ 2
+ = 
know that the addition of two matrices A and B is  3x x   8 8x   20 12x 
possible only if order of both the matrices A and B  2x 2 + 16 2x + 10x   2x 2 + 16 48 
⇒  = 
should be same.  3x + 8 x 2 + 8x   20 12x 
Hence, the sum of matrices A and b is not possible. [2] ⇒ 2x + 10x = 48
3 1 −1  2 1 −1 ⇒ 12x = 48
Q. 59. If X =   and Y =   , then find:
 5 − 2 −3  7 2 4  48
(i) X + Y ∴ x= =4 [2]
12
(ii) 2X − 3Y
Q. 61. Show by an example that for A ≠ 0, B ≠ 0, AB = 0.
(iii) a matrix Z such that X + Y + Z is zero matrix.
 [NCERT Exemp. Ex. 3.3, Q. 16, Page 54]
 [NCERT Exemp. Ex. 3.3, Q. 7, Page 53]
Ans. Given that, 0 −4  3 5
Ans. Let A=   ≠ 0 and B = 0 0  ≠ 0
3 1 −1  2 1 −1  0 2   
X=  and Y =  
(i)
5 −2 −3 2×3  7 2 4  2× 3 0 0 
∴ AB =  =0
3 + 2 1 + 1 −1 − 1  0 0 
X +Y =   Hence proved. [1]
5 + 7 −2 + 2 −3 + 4 
MATRICES | 73
Q. 62. Given Ans.
1 4 
2 4 0  2 8  is ( AB )′ = B′A′?  −1 0 −1  1 
A= and B =

3 9 6
  We have [2 1 3]  −1 1 0   0  = A
1 3 
 0 1 1   −1
 [NCERT Exemp. Ex. 3.3, Q. 17, Page. 54]
 −1 0 −1
1 4 
2 4 0   ∴ A = [2 1 3]  −1 1 0 
Ans. We have A =   and B =  2 8 
 3 9 6  2× 3  0 1 1 
1 3 
3× 2
= [ −2 − 1 + 0 0 + 1 + 3 −2 + 0 + 3]
 2 + 8 + 0 8 + 32 + 0  10 40 
∴ AB =  =  = [ −3 4 1]
3 + 18 + 6 12 + 72 + 18  27 102 
1
10 27  Now, A = [ −3 4 1]  0 
and ( AB)′ =   …(i)
 40 102    −1
 2 3 = [ −3 + 0 − 1]
1 2 1 4 9
Also, B′ =   and A′ =   = [ −4]
 4 8 3  2× 3  0 6 
3× 2 [2]
 2+8+0 3 + 18 + 6  10 27  4 0
∴ B′A′ =  = 0 −1 2   
+ 72 + 18  40 102 
…(ii) Q. 66. If A = 
 8 + 32 + 0 12
−  and B = 1 3  , then verify that:
  4 3 4   2 6 
Thus, we see that, (AB)′ = B′A′ (i) (A’)’ = A
[By using Eqs. (i) and (ii)] [2] (ii) (AB)’ = B’A’
Q. 63. Give an example of matrices A, B and C, such that
(iii) (kA)’ = (kA’)
AB =AC, where A is non-zero matrix but B ≠ C.
 [NCERT Exemp. Ex. 3.3, Q. 21, Page 55]  [NCERT Exemp. Ex. 3.3, Q. 27, Page 56]
1 0   2 3 2 3 4 0
 0 −1 2 
Ans. Let A =   , B =  4 0  and C =  4 4  [  B ≠ C ] Ans. We have, A =   and B= 1 3
 
 0 0      4 3 −4
   2 6 
1 0   2 3   2 3 
∴ AB =   =  …(i) (i) We have to verify that, (A′)′ = A
0 0   4 0   0 0  
0 4
1 0   2 3   2 3 
and AC =  ⋅ =  …(ii) ∴ A =  −1 3 

0 0   4 4   0 0    2 −4 
Thus, we see that AB =AC [By using Eqs. (i) and (ii)]  0 −1 2 
Where, A is non-zero matrix, but B ≠ C . [2] and (A′)' =  =A
 4 3 −4 
 x 0 0  a 0 0 Hence proved.
Q. 64. If P =  0 y 0  and Q =  0 b 0  , then prove that: [2]
 0 0 z   0 0 c  (ii) We have to verify that, AB′ = B′A′

 xa 0 0  3 9 
∴ AB = 
 
PQ =  0 yb 0  = QP. 11 −15
 0 0 zc  3 11 
      ⇒ ( AB)′ =  
 [NCERT Exemp. Ex. 3.3, Q. 23, Page. 55] 9 −15
 x 0 0   a 0 0   xa 0 0  0 4
Ans. PQ =  0 y 0  0 b 0  =  0 yb 0  …(i) 4 1 2  
and B′A′ =   −1 3
 0 0 z  0 0 c   0 0 zc   0 3 6   2 −4 
 
 a 0 0   x 0 0   ax 0 0  3 11 
= 
and QP = 0 b 0   0 y 0  =  0 by 0  …(ii) 9 −15
0 0 c   0 0 z   0 0 cz  = ( AB)′
Thus, we see that, PQ = QP [By using Eqs. (i) and (ii)] Hence proved.
[2]
Hence proved. [2]
(iii) We have to verify that (kA)′ = (kA’ )
 −1 0 −1  1 
 0 −k 2k 
Q. 65. If [ 2 1 3]  −1 1 0   0  = A , Find A Now, ( kA ) =  
    4 k 3k −4 k 
 0 1 1   −1
 [NCERT Exemp. Ex. 3.3, Q. 24, Page 55]
74 | OSWAAL NCERT SOLUTIONS – Textbook + Exemplar – MATHEMATICS : Class-XII

0 4k  Q. 68. Show that A′A and AA′ are both symmetric


  matrices for any matrix A.
and ( kA )′ =  − k 3k 
 [NCERT Exemp. Ex. 3.3, Q. 29, Page 56]
 2k −4k  Ans. Let P = A′A
0 4k  ∴ P ′ = ( A′A)′
 
Also, kA′ =  −k 3k  = A′( A′)′ [ ( AB′)′ = B′A′]
 2k −4k  = A′A = P
= ( kA )′ So, A′A is symmetric matrix for any matrix A.
Hence Proved. [2] Similarly,
 1 2  1 2 Let Q = AA′
    Q ′ = ( AA′)′
Q. 67. If A = 4 1  and B =  6 4  then verify that:
5 6  7 3  = ( A′)′ A′
(i) (2A + B)’ = 2A’ + B’ = AA′
(ii) (A − B)’ = A’ − B’ =Q
 [NCERT Exemp. Ex. 3.3, Q. 28, Page 56] So, AA′ is symmetric matrix for any matrix A. [2]
Ans. We have, Q. 69. Let A and B be square matrices of the order 3 × 3.
1 2  1 2 Is (AB)2 = A2B2? Give reasons.
A =  4 1  and B =  6 4 
   [NCERT Exemp. Ex. 3.3, Q. 30, Page 56]
Ans. Since, A and B are square matrices of order 3 × 3
 5 6  7 3 
  AB2 = AB·AB
    = ABAB
 2 4  1 2 
    = AABB[∵ AB = BA]
(i) ( 2 A + B) =  8 2  +  6 4      = A2B2
10 12  7 3  So, AB2 = A2B2 is true when AB = BA. [1]
3 6 Q. 70. Show that if A and B are square matrices such that
AB = BA, then (A + B)2 = A2 + B2 + 2AB.
= 14 6 
 [NCERT Exemp. Ex. 3.3, Q. 31, Page 56]
17 15
Ans. Since, A and B are square matrices such that AB = BA
 3 14 17  ∴ (A + B)2 = (A + B)·(A + B)
and ( 2 A + B)′ =   = A2 + AB + BA + B2
6 6 15 
= A2 + AB + AB + B2[∵ AB = BA]
1 4 5  1 6 7  = A2 + 2AB + B2
Also, 2 A′ + B′ = 2  +
2 1 6   2 4 3  Hence proved.
3 14 17   1 2 4 0 2 0 
= Q. 71. Let A =  , B =   , and C =  .
6 6 15   −1 3   1 5  1 −2 
= ( 2 A + B)′ If a = 4, b = –2, then show that:
Hence proved. (a) A + (B + C) = (A + B) + C
[2] (b) A(BC) = (AB)C
1 2  1 2  (c) (a + b)B = aB + bB
(ii) ( A − B) =  4 1  −  6 4  (d) a(C – A) = aC – aA
   
 5 6  7 3  (e) (AT)T = A
(f) (bA)T = bAT
0 0
(g) (AB)T = BTAT
=  −2 −3 (h) (A – B)C = AC – BC
 −2 3  (i) (A – B)T = AT – BT
 [NCERT Exemp. Ex. 3.3, Q. 32, Page 56]
0 −2 −2  Ans. We have,
and ( A − B)′ = 
0 −3 3   1 2 4 0 2 0 
A=  , B=  and C =  
1 4 5  1 6 7   −1 3  1 5  1 −2 
Also, A′ − B′ =  − 
2 1 6 2 4 3 a = 4, b = −2
0 −2 −2 
(a) A + ( B + C ) = 
= 1 2 6 0
 +
0 −3 3   −1 3   2 3
= ( A − B)′
7 2
=
6 
Hence proved. [2]
1
MATRICES | 75
T
5 2 2 0   1 2 1 −1
and (A + B) + C =  + (e) AT =   = 2 3 
0 8   1 −2   −1 3   
7 2 Now,
=
1 6  T
1 –1  1 2
= A + (B + C ) (AT )T =   =  −1 3  = A
 2 3   
Hennce proved. [2]
 4 0  2 0  Hence proved.
(BC ) =  [2]
(b)   [2]
 1 5  1 −2   −2 −4 
T

8 0  (f) ( bA )T =   [ b = −2]
= 2 −6 
7 −10  −2 2
= 
1 2  8 0   −4 −6 
and A(BC ) = 
 −1 3 7 −10 1 −1
and AT = 
 8 + 14 0 − 20 2 3 
=   −2 2
 −8 + 21 0 − 30 ∴ bAT =  = ( bA )T
 −4 −6 
 22 −20 [2]
= Hence proved.
13 −30
1 2   4 0  1 2   4 0   4 + 2 0 + 10   6 10 
Also, ( AB) =  ⋅ (g) AB =  = =
 −1 3  1 5  
 −1 3  1 5   −4 + 3 0 + 15  −1 15
 6 10  6 −1
=  ( AB)T =  
 −1 15 10 15 
6 10  2 0  Now,
( AB)C = 
 −1 15  1 −2   4 1  1 −1  6 −1
BT A T =   =  = ( AB)
T

 22 −20  0 5  2 3  10 15 
=
13 −30 Hence proved.
[2]
= A ( BC )
 1 − 4 2 − 0   −3 2 
Hence proved. (h)           ( A − B) =  = 
 −1 − 1 3 − 5   −2 −2 
(c)
4 0
( a + b )B = ( 4 − 2)   [ a = 4, b = −2]  −3 2   2 0   −4 −4 
1 5  ( A − B)C =   =  …(i)
     −2 −2  1 −2   −6 4  
8 0 
= 
 2 10   1 2   2 0   4 −4 
Now, AC =   =  …(ii)
 aB + bB = 4B − 2B  −1 3  1 −2   1 −6
16 0  8 0 
= −   4 0   2 0  8 0 
 4 20   2 10         and BC =   =  …(iii)
8 0  1 5  1 −2  7 −10 
= 
 2 10   4 − 8 −4 − 0   By using 
∴AC − BC =    Eqs. (ii) and (iii)
= ( a + b )B 1 − 7 −6 + 10   
Hence proved.
[2]  −4 −4 
= 
2 − 1 0 − 2   −6 4 
(d) (C − A ) =  
1 + 1 −2 − 3 = ( A − B)C [By using Eq. (i)]
1 −2  Hence proved. [2]
= 
 2 −5 T
 1 − 4 2 − 0 T
(i) ( A − B)TT = 1 − 4 2 − 0 
 4 −8  ( A − B) =  −1 − 1 3 − 5
 
and a(C − A ) =   [ a = 4]  −1 − 1 3T− 5 
8 −20  −−33 22 T
= 
8 0   4 8   4 −8  =  −2 −2
Also, aC − aA =  − =  
  −2 −2 
 4 −8  −4 12  8 −20   − 3 −2 
=  −3 −2 
= a(C − A ) =  2 −2
 2 −2 
Hence proved.  
[2]
−x2 0 
= 
 0 – x2 
and B = B ⋅ B
2

76 | OSWAAL NCERT SOLUTIONS – Textbook + Exemplar – MATHEMATICS : Class-XII = 0 1 0 1


  
1 0 1 0
1 −1  4 1  1 0
A T − BT =  − = 
2 3   0 5 0 1
 −3 −2  −x2 + 1 0 
= Now, A 2 + B2 =  
−2  − x + 1
2
2  0
= ( A − B)T 1 − x 2 0 
= 
Hence proved. [2]  0 1 − x2 
 cos θ sin θ  = ( A + B )2 [By using Eq. (i)]
Q. 72. If A =  then show that
 −sin θ cos θ Hence proved.
[2]
 cos 2θ sin 2θ   0 1 −1
A2 = 
 −sin 2θ cos 2θ Q. 74. Verify that A2 = I, when A =  4 −3 4  .
.
 [NCERT Exemp. Ex. 3.3, Q. 33, Page 57]  3 −3 4 
Ans. We have,  [NCERT Exemp. Ex. 3.3, Q. 35, Page 57]
 cos θ sin θ  Ans. We have,
A= 
 − sin θ cos θ  0 1 −1
A2 = A ⋅ A A =  4 −3 4 
 cos θ sin θ   cos θ sin θ   3 −3 4 
= ⋅ 
 − sin θ cos θ  − sin θ cos θ  0 1 −1  0 1 −1
 cos2 θ − sin 2 θ cos θ.sin θ + sin θ cos θ    
= A =  4 −3 4  ⋅  4 −3 4  [  A 2 = A ⋅ A]
2

 − sin θ cos θ − cos θ sin θ − sin 2 θ + cos2 θ   3 −3 4   3 −3 4 
 cos 2θ 2 sin θ cos θ  cos2 q − sin 2 q 
=   1 0 0 
 −2 sin θ cos θ cos 2θ   = cos 2q  = 0 1 0 
 cos 2θ sin 2θ  0 0 1 
=  [  sin2q = 2sinq ⋅ cos q ]
 − sin 2θ cos 2θ =I
Hence proved. [2]
[2] Hence proved.
0 −x 0 1  2
Q. 73. If A =   , B = 1 0  and x = −1, then show Q. 75. Find the values of a, b, c and d, if
 x 0   
a b  a 6   4 a+b
that (A + B)2 = A2 + B2 . 3  =  −1 2d  +  c+ d
 c d     3 
 [NCERT Exemp. Ex. 3.3, Q. 34, Page 57]
Ans. We have,  [NCERT Exemp. Ex. 3.3, Q. 41, Page 58]
Ans. We have,
 0 −x  0 1 
A=  ,B =   and x = −1
2
a b  a 6   4 a + b
x 0  1 0  3 = +
c d   −1 2d   c + d 3 
 0 − x + 1
∴( A + B ) =   3a 3b   a + 4 6 + a + b
 x + 1 0  ⇒  =
 3c 3d   c + d − 1 3 + 2d 
and
⇒ 3a = a + 4
 0 − x + 1  0 − x + 1
( A + B )2 =   x + 1 ∴ a=2
 x + 1 0  0 
⇒ 3b = 6 + a + b
1 − x 2 0  ⇒ 3b − b = 8 [ a = 2]
=  …(i)
 0 1 − x2  ∴ b=4
Also, A 2 = A ⋅ A 3d = 3 + 2d
∴ d=3
0 −x 0 −x
=   ⇒ 3c = c + d − 1
x 0  x 0 
⇒ 2c = 3 − 1
−x2 0 
= 2
∴ c =1
 0 –x  Thus, we have
and B2 = B ⋅ B
[2] a = 2 , b = 4 , c = 1 and d = 3
0 1 0 1
= 1 2 
1 0 1 0 Q. 76. If A =   then find A2+ 2A +7I.
4 1 
1 0  [NCERT Exemp. Ex. 3.3, Q. 43, Page 58]
=
0 1
−x2 + 1 0 
Now, A 2 + B2 =  
 0 − x 2
+ 1
2
Matrices | 77
Ans. We have, a = −2, c = −3 and b = −b
1 2 ⇒b = 0
A=
4 1  ∴ a = −2, b = 0 and c= − 3 [2]
1 2  1 2  Q. 79. If A is square matrix such that A2 = A, show that
A2 =  [  A 2 = A ⋅ A]
4 1   4 1  (I + A)3 = 7A + I.
 [NCERT Exemp. Ex. 3.3, Q. 47, Page 58]
 1 + 8 2 + 2
=  Ans. Since, A2 = A
4 + 4 8 + 1 
and ( I + A ) ⋅ ( I + A ) = I 2 + IA + AI + A 2
9 4
= = I 2 + 2 AI + A 2
8 9 
= I + 2 A + A = I + 3A
94   2 4  7 0 
A 2 + 2 A + 7I =  + + and ( I + A ) ⋅ ( I + A )( I + A ) = ( I + A )( I + 3A )
89  8 2   0 7 
= I 2 + 3 AI + AI + 3 A 2
18 8 
=  = I + 4 AI + 3 A
16 18 [2] = I + 7A = 7A + I
 cosα sinα  Hence proved. [2]
−1
Q. 77. If A =   and A = A’, then find the
 −sinα cosα  Q. 80. If A, B are square matrices of same order and B is
value of α. a skew-symmetric matrix, show that A′BA is skew
 [NCERT Exemp. Ex. 3.3, Q. 44, Page 58] symmetric.
Ans. We have,  [NCERT Exemp. Ex. 3.3, Q. 48, Page 58]
Ans. Since, A and B are square matrices of same order
 cos a sin a 
A=  and B is a skew-symmetric matrix, i.e., B’ = −B.
 − sin a cos a 
Now, we have to prove that A’BA is a skew-
cos a − sin a  symmetric matrix.
and A′ =  
 sin a cos a  ( A′BA )' = A′BA′
Also, A −1 = A′ = BA′A′ [ AB′ = B′ A′]
= A ′B ′ A
⇒ AA −1 = AA′
= A′( − BA )
 cosa sin a  cosa − sin a 
⇒ I=   = − A′BA
 − sin a cosa   sin a cosa 
Hence, A’BA is a skew-symmetric matrix. [2]
1 0  cos2 a + sin 2 a 0 
⇒  =   0 a −3 
0 1   sin 2 a + cos2 a 
Q. 81. If matrix A =  2 0 −1 is skew symmetric, find
0
By using equality of matrices, we get  b 1 0 
cos2 a + sin 2 a = 1 the value of ‘a’ and ‘b’.
Which is true for all real values of α. [2]  [CBSE Board, Delhi Region, 2018]
0 a 3  Ans. Given that,
Q. 78. If matrix  2 b -1 is a skew-symmetric matrix,  0 a −3
 c 1 0  A =  2 0 −1
then find the values of a, b and c.  b 1 0 
 [NCERT Exemp. Ex. 3.3, Q. 45, Page 58] We know that if A is a skew symmetric matrix then,
Ans.  Let, AT = –A
0 a 3   0 2 b  0 a −3
 a 0 1 = −  2 0 −1
A =  2 b −1    
 c 1 0   −3 −1 0  b 1 0 
Since, A is skew-symmetric matrix.  0 2 b   0 − a 3
∴ A′ = − A ⇒  a 0 1 =  −2 0 1
0 2 c  0 a 3   −3 −1 0  − b −1 0
⇒  a b 1  = −  2 b −1
 
∴− a = 2 and − b = −3
 3 −1 0   c 1 0 
⇒ a = −2 and b = 3 [1]
0 2 c   0 − a −3  2 −3  –1
Q. 82. Given, A =   . Compute A and show that
⇒  a b 1  =  −2 −b +1  −4 7 
 3 −1 0   − c −1 0  2A–1 = 9I – A.
By equality of matrices, we get  [CBSE Board, Delhi Region, 2018]
78 | OSWAAL NCERT SOLUTIONS – Textbook + Exemplar – MATHEMATICS : Class-XII

Ans. Given that,  0 a b


 
 2 −3 A =  −a 0 c 
A= 
 −4 7   −b − c 0 
1 7 3  A = − a( bc ) + a( bc )
⇒ A −1 =  
14 − 12  4 2  =0
1 7 3  Method 2 : Since A is a skew-symmetric matrix
=  
2 4 2 ∴ AT = − A
Now, ∴ AT = − A
−1
LHS = 2 A
= ( −1)3 ⋅ A
 1  7 3 
= 2    ⇒ A =− A
 2  4 2
⇒ 2 A = 0 or A = 0.
7 3 [2]
=  Q. 86. If A and B are square matrices of order 3 such that
4 2
|A| = –1, |B| = 3, then find the value of |2AB|.
RHS = 9I − A
 [CBSE Board, Foreign Scheme, 2017]
1 0   2 −3 Ans. We know that,
= 9 − 
0 1   −4 7             |A| = −1 and |B| = 3
 9−2 0 − ( −3) |2AB| = 23 |AB| = 8 × |A| × |B|
=  So,
 0 − ( −4 ) 9−7 
   |2AB|= 8 |A| × |B| = 8 × (−1) × 3= −24 [2]
7 3  Q. 87. If A is a square matrix such that A2 = I, then find
= 
4 2 the simplified value of (A – I)3 + (A + I)3 – 7A.
Since, LHS = RHS  [CBSE Board, Delhi Region, 2016]
Hence proved. [2] Ans. Given that,
Q. 83. If A is a 3 × 3 invertible matrix, then what will be ( A − I )3 + ( A + I )3 − 7 A
the value of k if det(A–1) = (det A)k. = A 3 − I 3 − 3 A 2 I + 3 AI 2 + A 3 + I 3 + 3 A 2 I + 3 AI 2 − 7 A
 [CBSE Board, Delhi Region, 2017] = 2 A 3 + 6 AI 2 − 7 A
AdjA = 2 A ⋅ A 2 + 6 AI 2 − 7 A
Ans. A −1 =
A = 8A − 7 A
3 −1
=A [1]
−1 AdjA A
∴A = =  0 2b −2 
A A
Q. 88. Matrix A =  3 1 3  is given to be symmetric,
[∵ If A is a non-singular matrix of order n, then
 3a 3 −1
|adj(A)| = |A|n-1]
A
2 find values of a and b.
=  [CBSE Board, Delhi Region, 2016]
A
Ans. We have,
As we are given that |A−1| = |A|k  0 2b −2 
∴                k = –1 [1] A =  3 1 3 
Q. 84. Show that all the diagonal elements of a skew 3a 3 −1
symmetric matrix are zero.
 [CBSE Board, Delhi Region, 2017] It is given that the matrix is symmetric.
Ans. Let A = [aij]n × n be skew symmetric matrix. ∴ A = A′
A is skew symmetric matrix.  0 2b −2   0 3 3a 
∴ A = −A    
⇒  3 1 3  =  2b 1 3 
⇒ aij = a ji ∀i , j 3a 3 −1  −2 3 −1
For diagonal elements i = j, Now, by equality of matrices, we get:
⇒ 2aii = 0 2b = 3
⇒ 2ii = 0 3
⇒ b=
Thus, diagonal elements are zero. 2
[2]
and 3a = −2
Q. 85. If A is a skew-symmetric matrix of order 3, then
prove that det A = 0. −2
⇒ a=
 [CBSE Board, All India Region, 2017] 3
Ans. Method 1 : Any skew symmetric matrix of order 3 −2 3
Therefore, a = and b =
is: 3 2 [1]
1 3]1×3  ( −1) × 1 + 1 × 0 + 0 × ( −1) =A
[2 
 0 × 1 + 1 × 0 + 1 × ( −1) 
 −1 + 0 + 1 
[2 1 3]1×3  −1 + 0 + 0  = A MATRICES | 79
 0 + 0 − 1 
Q. 89. Write the number of all possible matrices of order 0
2 × 2 with each entry 1, 2 or 3.
 [CBSE Board, All India Region, 2016]
[ 2 1 3]1×3  −1 = A
 −1
Ans. Given that,
Order is 2 × 2. [ 2 × 0 + 1 × ( −1) + 3 × ( −1)] = A
Number of elements = 4 in which 1, 2 or 3 (Three) [0 − 1 − 3] = A
entry can be filled and can repeat.
[ −4]1×1 = A
Number of possible matrices = 34 = 81 [1]
The order of the matrix will be 1 × 1. [1]
Q. 90. Use elementary column operation C2 → C2
+ 2C1 in the following matrix equation :  3 5
Q. 92. If A =   is written as A = P + Q, where P
 2 1  3 1 1 0  7 9
 =   is a symmetric matrix and Q is skew symmetric
 2 0   2 0   −1 –1  .
matrix, then write the matrix P.
 [CBSE Board, All India Region, 2016]  [CBSE board, Foreign Scheme, 2016]
Ans. By using this operation, C2→C2+2C1 on given Ans. We know that,
matrices, we get
1
 2 5   3 1  1 2  P = ( A + A′)
 =   2
 2 4   2 0  −1 −1 [1]
Given that,
 −1 0 −1  1   3 5
   A= 
Q. 91. If (2 1 3)  −1 1 0  0  = A then write the 7 9 
 0 1 1  −1 
   3 7 
A′ =  
order of matrix A. 5 9 
 [CBSE Board, Foreign Scheme, 2016] Now,
Ans.
 −1 0 −1  1  1   3 5 3 7  
P=  + 
2  7 9  5 9  
[ 2 1 3]1×3  −1 1 0   0  = A
 0 1 1   −1 1 3 + 3 5 + 7
= 
2 7 + 5 9 + 9 
3× 3

( −1) × 1 + 0 × 0 + ( −1) × ( −1)


[ 2 1 3]1×3  ( −1) × 1 + 1 × 0 + 0 × ( −1)  = A =
1  6 12 
 0 × 1 + 1 × 0 + 1 × ( −1)  2 12 18
 −1 + 0 + 1   3 6
= 
[ 2 1 3]1×3  −1 + 0 + 0 = A 6 9
 0 + 0 − 1  Hence proved. [1]
0
[2 1 3]1×3  −1 = A
Short Answer Type Questions
 −1
(3 or 4 marks each)
[ 2 × 0 + 1 × (a−12)×+ 23 ×matrix,
Q. 1. Construct
( −1)] = A
A = [aij], whose elements  9
[0 −given
are 1 − 3] =by:
A 2 2 
Therefore, the required matrix is A =   [1]
[ −4] (i=+ Aj )2 i 9 8 
(i) aij =1×1 (ii) aij = i
2 j (ii) aij = , i , j = 1, 2  2 
j
(i + 2 j)2
(iii) aij =  [NCERT Ex. 3.1, Q. 4, Page 64] 1 1
2 ∴ a11 = = 1 a12 =
1 2
Ans. In general, a 2 × 2 matrix is given by
2 2
a a  a21 = = 2 a22 = = 1
A =  11 12  1 2  1
1
 a21 a22  Therefore, the required matrix is A =  2 [1]
 
(i + j ) 2 (i + 2 j ) 2  2 1 
(i) aij = ; i , j = 1, 2 (iii) aij = , i , j = 1, 2
2 2
Therefore,
(1 + 2)2 32 9 (1 + 4)2 52 25
(1 + 1)2 4 (1 + 2)2 9 ∴ a11 = = = a12 = = =
a11 = = = 2 a12 = = 2 2 2 2 2 2
2 2 2 2
( 2 + 2 )2 4 2 ( 2 + 4 )2 6 2
( 2 + 1)2 9 ( 2 + 2)2 16 a21 = = = 8 a22 = = = 18
a21 = = a22 = = =8 2 2 2 2
2 2 2 2
80 | OSWAAL NCERT SOLUTIONS – Textbook + Exemplar – MATHEMATICS : Class-XII

 9 25  x z 1 −1  3 5
Ans. 2  + 3 0  = 3
Therefore, the required matrix is A =  2 2  [1]
  y t  2 4 6 
8 18   x
2 2 z  3 −3  9 15
⇒  + =
Q. 2. Find the values of x, y and z from the following
2y 2t  0 6  12 18
equations:
 2x + 3 2 z − 3 9 15
 4 3  y z   x + y 2  6 2  ⇒  = 
(i)   = 1 5  (ii)  =  2y 2t + 6  12 18
 x 5    5 + z xy  5 8  [1]
The corresponding elements of these two matrices,
 x + y + z  9 
    we get
(iii)  x + z  = 5  2x + 3 = 9
  y + z  7  [NCERT Ex. 3.1, Q. 6, Page 64] ⇒    2x = 6
Ans. ⇒    x = 3
 4 3  y z           2y = 12
(i)  = 
 x 5 1 5  ⇒          y = 6
As the given matrices are equal, their corresponding     2z – 3 = 15
elements are also equal. ⇒      2z = 18
Comparing the corresponding elements, we get: ⇒  z = 9
x = 1, y = 4 and z = 3 [1]   2t + 6 = 18
 x + y 2  6 2 ⇒    2t = 12
(ii)  =  ⇒       t = 6
5 + z xy  5 8 
∴    x = 3, y = 6, z = 9 and t = 6 [2]
As the given matrices are equal, their corresponding
2   −1  10 
elements are also equal. Q. 4. If x   + y   =   . Find values of x and y.
Comparing the corresponding elements, we get: 3   1  5 
x + y = 6, xy = 8, 5 + z = 5  [NCERT Ex. 3.2, Q. 11, Page 81]
⇒ x – y = ±2 2  −1 10 
Ans. x  + y  =  
Now, when x – y = 2 and x + y = 6, we get x = 4 3   1 5 
and y = 2  2x   − y  10 
When x – y = –2 and x + y = 6, we get x = 2 and y = 4 ⇒  + = 
3x   y  5 
∴ x = 4, y = 2, and z = 0
 2x − y  10 
or ⇒ 3 +  = 5  [1]
 x y  
x = 2, y = 4, and z = 0 [1]
Comparing the corresponding elements of these
 x + y + z  9  two matrices, we get:
(iii)  x + z  = 5  2x – y = 10 and 3x + y = 5
 y + z  7  Adding these two equations, we have:
  5x = 15 ⇒ x = 3
As the two matrices are equal, their corresponding
Now, 3x + y = 5
elements are also equal. ⇒                               y = 5 – 3x
Comparing the corresponding elements, we get: ⇒                               y = 5 – 9 = –4
x + y + z  = 9 … (i) ∴                                x = 3 and y= –4. [2]
               x + z = 5 … (ii)  x y   x 6  4 x + y
Q. 5. Given 3  = + . Find
               y + z = 7 … (iii)  z w   −1 2 w   z + w 3 
From equations (i) and (ii), we have: the values of x, y, z and w.
 [NCERT Ex. 3.2, Q. 12, Page 81]
y+5=9
 x y   x 6  4 x + y
     ⇒ y = 4 Ans. 3  =  −1 2  +  +
z w  w z w 3 
Then, from equation (iii), we have:
  4 + z = 7 3x 3y   x + 4 6+x+ y
⇒  = − + +  [1]
⇒       z = 3 3z 3w   1 z w 2w + 3 
∴ x + z= 5 Comparing the corresponding elements of these
⇒       x = 2 two matrices, we get:
   3x = x + 4
∴     x = 2, y = 4, and z = 3 [1]
⇒ 2x = 4
Q. 3. Solve the equation for x, y, z and t if ⇒     x = 2
 x z 1 −1 3 5            3y = 6 + x + y
2  + 3 0 2  = 3  4 6  ⇒ 2y = 6 + x = 6 + 2 = 8
 y t     
⇒ y=4
 [NCERT Ex. 3.2, Q. 10, Page 81]
        3w = 2w + 3
MATRICES | 81
⇒   W = 3 Comparing the corresponding elements, we have:
            3z = –1 + z + w 3k − 2 = 1
⇒   2z = –1 + W = –1 + 3 = 2 ⇒ 3k = 3
⇒       z = 1
∴        x = 2, y = 4, z = 1, and W = 3 [2] ⇒ k =1
cos x − sin x 0  Thus, the value of k is 1. [3]
Q. 6. If F ( x ) =  sin x cos x 0  . Show that . Q. 8. The bookshop of a particular school has 10 dozen
0 0 1  chemistry books, 8 dozen physics books, 10 dozen
F ( x) F ( y) = F ( x + y) economics books. Their selling prices are `80, `60

and `40 each respectively. Find the total amount
 [NCERT Ex. 3.2, Q. 13, Page 82]
cos x − sin x 0  the bookshop will receive from selling all the
Ans.  0  books using matrix algebra.
F( x ) = sin x cos x
0 0 1   [NCERT Ex. 3.2, Q. 20, Page 83]
Ans. The bookshop has 10 dozen chemistry books, 8
cos y − sin y 0 
 dozen physics books, and 10 dozen economics
F( y ) = sin y cos y 0 
books. The selling prices of a chemistry book,
0 0 1 
a physics book, and an economics book are
cos ( x + y ) − sin ( x + y ) 0  respectively given as ` 80, `60, and `40.
 
F( x + y ) = sin ( x + y ) cos ( x + y ) 0 The total amount of money that will be received
0 0 1 
 from the sale of all these books can be represented
cos x − sin x 0  cos y − sin y 0  in the form of a matrix as:
F( x )F( y ) = sin x cos x 0  sin y cos y 0  80 
 
0 0 1  0 0 1  12 [10 8 10] 60 
 − cos x sin y   40 
cos x cos y − sin x sin y + 0 − sin x cos y + 0 0
  = 12[10 × 80 + 8 × 60 + 10 × 40]
 − sin x sin y  = 12 (800 + 480 +400)
= sin x cos y + cos x sin y + 0 0
 + cos x cos y + 0  = 12(1680)
0 0 1 = 20160 [3]
  1 1
  Q. 9. Find (A + A′) and (A – A′) when
2 2
cos ( x + y ) − sin ( x + y ) 0
   0 a b 
= sin ( x + y ) cos ( x + y ) 0 A = − a 0 c .
[3]  − b − c 0 
0 0 1 
 [NCERT Ex. 3.3, Q. 9, Page 89]
= F (x + y) Ans. Given that,
∴ F( x )F( y ) = F( x + y )  0 a b
 
3 −2  1 0  2
A =  −a 0 c 
Q. 7. If A =   and I =   . Find k so that A =
 4 −2  0 1   −b − c 0 
kA – 2I. [NCERT Ex. 3.2, Q. 17, Page 82]  0 − a −b 
Ans.         A = A ⋅ A
2
∴ A ' =  a 0 − c 
3 −2  3 −2  b c 0 
=  
 4 −2   4 −2  Now,
3(3) + ( −2)( 4) 3( −2) + ( −2)( −2) 
=   0 a b   0 − a −b 
 4(3) + ( −2)( 4) 4( −2) + ( −2)( −2) A + A ' =  − a 0 c  +  a 0 − c 
1 −2 
=   −b − c 0  b c 0 
 4 −4 
2
Now          A = kA – 2I 0 + 0 a − a b − b 
1 −2  3 −2  1 0  =  − a + a 0 + 0 c − c 
⇒  = k  4 −2  − 2 0 1   −b + b − c + c 0 + 0 
 4 − 4     
1 −2  3k −2k   2 0  0 0 0 
⇒ = − 
 4 −4   4k −2k  0 2  = 0 0 0 
1 −2  3k − 2 − 2k  0 0 0 
⇒  =
 4 −4   4k −2k − 2 
Therefore,
1
Skew symmetric matrix = ( A − A ')
2
1  3 5  3 1  
82 | OSWAAL NCERT SOLUTIONS – Textbook + Exemplar – MATHEMATICS : Class-XII =   − 
2  1 −1  5 −1 
0 0 0  1  0 4
1 1  =  
( A + A ') = 0 0 0  2  −4 0 
2 2
0 0 0   0 2
= 
0 0 0   −2 0 
= 0 0 0  Therefore, given matrix A is sum of symmetric
0 0 0  3 3 
matrix   and skew symmetric matrix
Now, 3 −1
 0 a b   0 − a −b   0 2
     −2 0  . [3]
A − A ' =  −a 0 c  − a 0 −c   
 −b −c 0  b c 0  (ii) Given that,
 0 − 0 a + a b + b  6 −2 2
=  −a − a 0 − 0 c + c  A =  −2 3 −1 
 −b − b − c − c 0 − 0   2 −1 3 
 0 2a 2b   6 −2 2
∴ A ' =  −2 
=  −2a 0 2c 
 3 −1 
 −2b −2c 0   2 −1 3 

Thus, 1
Symmetric matrix = ( A + A ')
 0 2 a 2b  2
1 1
( A − A ') =  −2a 0 2c    6 −2 2   6 −2 2  
1 
2 2
 −2b −2c 0  =   −2 3 −1  +  −2 3 −1  
2
  2 −1 3  2 −1 3 
 0 −a b 
=  − a 0 c  12 −4 4 
1
=  −4 6 −2 
 −b − c 0  [3] 2
 4 −2 6 
Q. 10. Express the following matrices as the sum of a
symmetric and skew symmetric matrix :  6 −2 2 
 6 −2 2 =  −2 3 −1 
3 5   
(i)   (ii)  −2 3 −1  2 −1 3
1 −1
 2 −1 3  1
 3 3 −1  Skew symmetric matrix = ( A − A ')
1 5  2
(iii)  −2 −2 1 (iv)     6 −2 2   6 −2 2 
  −1 2  1
 −4 −5 3 =   −2 3 −1 −  −2 3 −1 
 [NCERT Ex. 3.3, Q. 10, Page 89] 2   2 −1 3   2 −1 3  
   
Ans. (i) Given that,
3 5 1 0 0 0 
= 0 0 0 
A =  2 0 0 0 
1 −1   
3 1  0 0 0 
∴ A' =   = 0 0 0 
5 −1 0 0 0 
 
1
Symmetric matrix = ( A + A ') Therefore, given matrix A is sum of symmetric
2  6 −2 2 
1  3 5  3 1   matrix  −2 3 −1 and skew symmetric matrix
=   +   2 −1 3 
2  1 −1  5 −1   
1 6 6 0 0 0 
=   0 0 0 
2 6 −2  0 0 0 
 . [3]
3 3 
=  (iii) Given that
3 −1
 3 3 −1
1  
Skew symmetric matrix = ( A − A ') A =  −2 −2 1
2  −4 −5 2 
1  3 5  3 1  
=   − 
2  1 −1  5 −1 
1  0 4
=  
2  −4 0 
 0 2
=
MATRICES | 83

 3 −2 −4  1
Symmetric matrix = ( A + A ')
  2
A ' =  3 −2 −5 
 −1 1 2  1   1 5  1 −1 
=   + 
2   −1 2  5 2  
1
Symmetric matrix = ( A + A ')
2 1 2 4
=  
  3 3 −1   3 −2 −4   2 4 4
1 
=   −2 −2 1  +  3 −2 −5   1 2 
2 = 
  −4 −5 2   −1 1 2   2 2
 6 1 −5  1
1 Skew symmetric matrix = ( A − A ')
=  1 −4 −4  2
2
 −5 −4 4  1   1 5  1 −1 
=   − 
 1 5 2   −1 2  5 2  
 3 −
2 2  0 3
  =
1 
= −2 − 2  −3 0 
 2 
  Therefore, given matrix A is sum of symmetric
− 5 −2 2
 2  1 2 
matrix   and skew symmetric matrix
1 2 2
Skew symmetric matrix = ( A − A ')
2  0 3
  3 3 −1  3 −2 −4    . [3]
 −3 0 
1 
=   −2 −2 1 −  3 −2 −5   Q. 11. Find the inverse of each of the matrices, if it exists
2
  −4 −5 2   −1 1 2   2 3 
in given matrices is  
 0 5 3 5 7 
1   [NCERT Ex. 3.4, Q. 4, Page 97]
=  −5 0 6
2 2 3 
 −3 −6 0 
Ans. Let A =  
 5 3 5 7 
 0 2 2 We know that A = IA
 
5  2 3  1 0 
= − 0 3 ∴  = A
 2  5 7   0 1 
 
− 3 −3 0  3 1 
1 0  1 
 2  ⇒  2 = 2 A  R1 → 2 R1 
     
Therefore, given matrix A is sum of symmetric 5 7  0 1 
 1 5  3  1 
 3 −  1   2 0
2 2 2
  ⇒ = A ( R2 → R2 − 5R1 )
 1 −2 − 2  0 − 1   − 5 1 
matrix   and skew symmetric
2  2   2 
 
 − 5 −2 2 1 0   −7 3
 2  ⇒ = A ( R1 → R1 + 3R2 )
0 − 1   − 5 1 
 5 3  2   2 
 0 2 2
  1 0   −7 3 
5 ⇒  = A ( R2 → −2R1 )
matrix  − 0 3. [3] 0 1   5 −2 
 2 
   −7 3 
− 3 ∴ A −1 =  
−3 0   5 −2 
 2  [3]
(iv) Given that, Q. 12. Find the inverse of each of the matrices, if it exists
2 1 
 1 5 in given matrices  
A= 7 4 
 −1 2 
 [NCERT Ex. 3.4, Q. 5, Page 97]
1 −1 2 1 
∴ A' =   Ans. Let A = 
5 2 
7 4
We know that A = IA
3 1  1 0 
∴   = A 
5 2  0 1 
84 | 1 1  : Class-XII
OSWAAL NCERT SOLUTIONS – Textbook + Exemplar – MATHEMATICS
⇒  = A
 1 0
( C1 → C1 − 2C2 )
 
1 2   −2 1 
2 1  1 0 1 0  1 −1
∴ 7 =
4  0 1 
A ⇒   = A  ( C2 → C2 − C1 )
 1 1  −2 3 
 1 1  1 0  2 −1 
1 2 = 2
0  1  ⇒ = A ( C1 → C1 − C2 )

   
A  R1 → 2 R1  0 1   −5 3
 
7 4  0 1 
 2 −1
 1  1  ∴ A −1 =  
1 0  −5 3 
2 =  2 [3]
⇒    A ( R2 → R2 − 7R1 ) Q. 15. Find the inverse of each of the matrices, if it exists
0 1  7
− 1
 2   2  4 5 
in given matrices is  
1 0   4 −1  3 4 
⇒  1  =  7 A ( R1 → R1 − R2 )  [NCERT Ex. 3.4, Q. 8, Page 97]
0 − 1
 2  2  4 5 
Ans. Let A =  
1 0  4 −1  3 4 
⇒ 0 = A ( R2 → 2R2 )
 1   −7 2  4 5  1 0 
∴  = A
∴ A −1 = 
 4 −1
3 4  0 1 
 −7 2  1  1 −1
[3] 1
⇒ = A ( R1 → R1 − R2 )
Q. 13. Find the inverse of each of the matrices, if it exists 3 4  0 1
2 5 1 1  1 −1
in given matrices is  . ⇒  = A ( R2 → R2 − 3R1 )
1 3  0 1  −3 4 
 [NCERT Ex. 3.4, Q. 6, Page 97]
1 0   4 −5 
 2 5 ⇒ = A ( R1 → R1 − R2 )
Ans. Let A =   0 1   −3 4 
1 3
 4 −5
We know that A = IA ∴ A −1 =  
 2 5 1 0  −3 4  [3]
∴  = A
1 3 0 1  Q. 16. Find the inverse of each of the matrices, if it exists
 5 1  3 10 
1 0  1  in given matrices is  .
⇒  2 = 2 A  R1 → 2 R1  2 7 
     
 1 3   0 1   [NCERT Ex. 3.4, Q. 9, Page 97]
 5  1   3 10 
Ans. Let A= 
1 2   2 0 2 7 
⇒ = A ( R2 → R2 − R1 ) We know that A =IA
0 1   − 1 1 
 2   2  3 10  1 0 
∴  = A
1 0   3 −5  2 7  0 1 
⇒ 1  =  1 A ( R1 → R1 − 5R2 ) 3  1 −1
0 − 1 1
⇒  = A ( R1 → R1 − R2 )
 2  2 
2 7  0 1
1 0  3 −5 
⇒ = A ( R2 → 2R2 ) 1 3  1 −1
0 1   −1 2  ⇒  = A ( R2 → R2 − 2R1 )
0 1   −2 3 
 3 −5
∴ A −1 =   1 0   7 −10 
 −1 2  ⇒  = A ( R1 → R1 − 3R2 )
[3] 0 1   −2 3 
Q. 14. Find the inverse of each of the matrices, if it exists  7 −10 
3 1  ∴ A −1 =  
in given matrices is  .  −2 3
[3]
5 2 
Q. 17. Find the inverse of each of the matrices, if it exists
 [NCERT Ex. 3.4, Q. 7, Page 97]
 3 −1 
3 1  in given matrices is  .
Ans. Let A =    −4 2 
5 2 
We know that A = AI  [NCERT Ex. 3.4, Q. 10, Page 97]
3 1  1 0   3 −1 
Ans. Let A = 
∴   = A   −4 2 
5 2  0 1 
1 1   1 0 We know that A = AI
⇒   = A  ( C1 → C1 − 2C2 )
1 2   −2 1 
1 0   1 −1
⇒   = A  ( C2 → C2 − C1 )
1 1   −2 3 
1 0   2 −1 
⇒  = A  −5 ( C1 → C1 − C2 )
 0 1   3
MATRICES | 85

 3 −1 1 0  Q. 20. Find the inverse of each of the matrices, if it exists


∴   = A  2 −3 
 −4 2  0 1  in given matrices is  .
1 −1  1 0   −1 2 
⇒ 
2 
= A ( C1 → C1 + 2C2 )
0  2 0   [NCERT Ex. 3.4, Q. 13, Page 97]
1 0  1 1   2 −3 
⇒   = A ( C2 → C2 + C1 ) Ans. Let A = 
2 
 0 2   2 3  −1
 1 We know that A = AI
1 0  1 2   1   2 −3  1 0 
⇒  = A  C2 → 2 C2 
0 1  3 ∴  = A
2   
 −1 2  0 1 
 2
 1 −1  1 1
 1 ⇒  = A ( R1 → R1 + R2 )
−1
1 2   −1 2  0 1
∴ A =
3 1 −1 1 1
2  ⇒  = A ( R2 → R2 + R1 )
 2 [3] 0 1 1 2 
Q. 18. Find the inverse of each of the matrices, if it exists 1 0   2 3
⇒ 0 1  = 1 A ( R1 → R1 + R2 )
 2 −6 
in given matrices is     2 
.
1 −2  2 3 
 [NCERT Ex. 3.4, Q. 11, Page 97] ∴ A −1 =  
1 2  [3]
 2 −6 
Ans. Let A =   Q. 21. Show that the matrix B′AB is symmetric or skew
1 −2  symmetric according as A is symmetric or skew
We know that A = AI
symmetric. [NCERT Misc. Ex. Q. 5 Page 100]
 2 −6  1 0  Ans. We suppose that A is a symmetric matrix, then
∴   = A
1 − 2  0 1  A′ = A … (i)
 2 0 1 3 Consider,
⇒  = A ( C2 → C2 + 3C1 )
1 1  0 1  ( B ' AB ) ' = {B ' ( AB )} '
 2 0  −2 3 = ( AB ) ' ( B ' ) ' ( AB ) ' = B ' A ' 
⇒  = A ( C1 → C1 − C2 )
0 1   −1 1  = B ' A ' (B) ( B ' ) ' = B 
 −1 3 = B ' ( A ' B)
1 0   1   1 
⇒  = A  C1 → 2 C1 
0 1  − 1   = B ' ( AB )  By using Eq. ( i ) 
 2 
∴ ( B ' AB ) ' = B ' AB
 −1 3
∴ A −1 =  1  Thus, if A is a symmetric matrix, then B′AB is a
− 1
symmetric matrix.
 2  [3]
Now, we suppose that A is a skew-symmetric
Q. 19. Find the inverse of each of the matrices, if it exists
matrix.
 6 −3 
in given matrices is  . Then,
 −2 1 A′= −A
 [NCERT Ex. 3.4, Q. 12, Page 97] Consider,
 6 −3 ( B ' AB ) ' = B ' ( AB ) '
Ans. Let A =  
 −2 1 = ( AB ) ' ( B ' ) '
We know that A = IA = (B ' A ') B
 6 −3 1 0  = B '( − A )B
∴  = A
 −2 1 0 1  = −B ' AB
Therefore,
 1  1 
1 −   0  1  ( B ' AB ) ' = −B ' AB
⇒ 2 = 6 A  R1 → R1 
    6 
 −2 1  0 1  Thus, if A is a skew-symmetric matrix, then B′AB is a
skew-symmetric matrix. Hence, if A is a symmetric
1 
 1 0 or skew-symmetric matrix, then B′AB is a symmetric
 1 −  6


2 =

A ( R2 → R2 + 2R1 ) or skew symmetric matrix accordingly. [3]
1
0 0   1 Q. 22. Find the values of x, y, z if the matrix
 3  0 2 y z
Now, in the above equation, we can see that all the A =  x y − z  . Satisfy the equation A′A = I.
zeros in the second row of the matrix on the LHS.
Therefore, A−1 does not exist. [3]  x − y z 
 [NCERT Misc. Ex. Q. 6, Page 100]
86 | OSWAAL NCERT SOLUTIONS – Textbook + Exemplar – MATHEMATICS : Class-XII

Ans. It is given that,  8 5  3 1 1 0 


 −5 3 − 5  −1 + 7
2 
0 2y z 
   0 1 
A =  x y − z 
8 5 15 5  7 0 
 x − y z  = − + 
       −5 3  −5 10  0 7 
 0 x x  −7 0 7 0 
 = +
∴ A ' = 2y y − y   0 −7  0 7 
 z − z z  0 0
= 
Now, A′A = I 0 0
0 x x  0 2 y z  1 0 0          = 0 = RHS

⇒ 2y 
y −y  x y − z  = 0 1 0 ∴ A2 – 5A + 7I = 0 [3]
 z − z z   x − y z  0 0 1  1 0 2   x 
  
0 + x + x
2 2
0 + xy − xy 0 − xz + xz  Q. 24. Find x, if [ x −5 −1] 0 2 1   4  = 0.
   2 0 3  1 
⇒ 0 + xy − xy 4 y + y + y 2 yz − yz − yz 
2 2 2

0 − xz + zx 2 yz − yz − yz z 2 + z 2 + z 2   [NCERT Misc. Ex. Q. 9, Page 100]


 
Ans. We have,
1 0 0
1 0 2   x 
= 0 1 0
0 0 1 
[ x − 5 − 1 ] 0 2 1   4  = 0
 2 0 3  1 
2x 2 0 0  1 0 0
  x 
⇒ 0 6y 2 0  = 0 1 0
⇒ [ x + 0 − 2 0 − 10 + 0 2x − 5 − 3]  4  = 0
0 0 3z 2  0 0 1 
 1 
On comparing the corresponding elements, we x 
have:
2x 2 = 1
⇒ [ x − 2 −10 2 x − 8 ]  4  = 0
1 
1
∴x = ±
2
⇒  x ( x − 2 ) − 40 + 2x − 8 = 0
6y 2 = 1 ⇒  x 2 − 2x − 40 + 2x − 8 = [ 0]

∴y = ±
1 ⇒  x 2 − 48 = [ 0]
6
∴ x 2 − 48 = 0
3z 2 = 1
⇒ x 2 = 48
1
∴z = ± ⇒ x = ±4 3
3 [3] [3]
 3 1 1 3 2  1 
2   
Q. 23. If A =   . Show that A – 5A + 7I = 0. Q. 25. Find the value of x, if [ 1 x 1]  2 5 1   2  = 0 .
  −1 2  [NCERT Misc. Ex. Q. 8, Page 100]
15 3 2   x 
Ans. It is given that,
 [NCERT Exemp. Ex. 3.3, Q. 10, Page 54]
 3 1
A=  Ans. We have,
      −1 2  1 3 2  1 
Therefore, [1 x 1 ]1×3  2 5 1   2  = 0
A2 = A ⋅ A 15 3 2   x 
3× 3 3×1
 3 1 3 1 1 
=  
 −1 2   −1 2  ⇒ [1 + 2x + 15 3 + 5x + 3 2 + x + 2]1×3  2  =0
 3(3) + 1( −1) 3(1) + 1( 2)   x 
=
(1) + 2( 2)
3×1

 −1( 3) + 2( −1) −1 1 
 9 − 1 3 + 2 ⇒ [16 + 2x 5x + 6 x + 4]1×3  2  =0
=   x 
 −3 − 2 −1 + 4  3×1

 8 5 ⇒ [16 + 2x + (5x + 6) ⋅ 2 + ( x + 4) ⋅ x ]1×1 = 0


= 
 −5 3 ⇒ 16 + 2x + 10x + 12 + x 2 + 4x  = 0
LHS = A – 5 A + 7 I
2
MATRICES | 87

⇒  x 2 + 16x + 28 = 0 4  −4 8 4 


   
⇒  x 2 + 2x + 14x + 28 = 0 Q. 27. Find A, if 1  A =  −1 2 1  .
3   −3 6 3 
⇒ ( x + 2)( x + 14) = 0
 [NCERT Exemp. Ex. 3.3, Q. 13, Page 54]
∴ x = −2, −14 [3] Ans. We have,
 5 3 4  −4 8 4 
Q. 26. Show that A =   satisfies the equation A2 1  A =  −1 2 1 
 −1 −2     
− 3A − 7I = 0 and hence find A−1. 3   −3 6 3 
3×1 3× 3
 [NCERT Exemp. Ex. 3.3, Q. 11, Page 54]
Ans. We have, Let A = [x y z]
 5 3 4  −4 8 4
∴ 1  [ x y z]1×3 =  −1
 
A=  2 1 
 −1 −2 
3   −3 6 3  3×3
3×1
A2 = A ⋅ A
 4x 4 y 4 z   −4 8 4
 5 3  5 3
=   ⇒  x y
 
z  =  −1 2 1

 −1 −2   −1 −2  3x 3y 3z   −3 6 3 
 25 − 3 15 − 6 
=  ⇒      4x = –4
 −5 + 2 −3 + 4  ⇒       x = –1
 22 9  ⇒       4y = 8
= 
 −3 1  ⇒      y   = 2
and 4z = 4
 5 3 ⇒    z = 1
3A = 3  
 −1 −2  ∴          A = [–1 2 1] [3]
15 9 3 −4 
=    2 1 2 
 −3 −6  Q. 28. If A = 1 1 and B =  , then verify
 2 0  1 2 4 
1 0 
7I = 7  
0 1  (BA)2 ≠ B2 A2.
7 0   [NCERT Exemp. Ex. 3.3, Q. 14, Page 54]
= 
0 7  Ans. We have,
Therefore, 3 −4 
2 1 2
 22 9  15 9  7 0  A = 1 1 and B =  
A 2 − 3A − 7I =  − −   2 0  1 2 4  2×3
 −3 1   −3 −6  0 7  3× 2

 22 − 15 − 7 9 − 9 − 0  3 −4 
= 2 1 2  
 BA =   1 1
 −3 + 3 − 0 1 + 6 − 7  1 2 4  2× 3  2 0 
0 0   3× 2
=  6 + 1 + 4 −8 + 1 + 0 
0 0 = 
=0 3 + 2 + 8 −4 + 2 + 0 
Hence proved. 11 −7 
=
Since, A − 3A − 7 I = 0
2
13 −2 
A −1[( A 2 ) − 3A − 7 I ] = A −1 0 11 −7  11 −7 
and ( BA ) ⋅ ( BA ) = 
−1 −1 −1
A A ⋅ A − 3 A A − 7 A I = 0 [Q A 0 = 0] −1
13 −2  13 −2 
121 − 91 −77 + 14 
IA − 3I − 7 A −1 = 0 [Q A −1 A = I ] ( BA )2 =  
−1 143 − 26 −91 + 4 
A − 3I − 7 A = 0 [Q A −1I = A −1 ]
30 −63 
− 7 A −1 = − A + 3I = 
117 −87  …(i)
 −5 −3 3 0 
= +  2 1 2 2 1 2
 1 2  0 3  Also, B 2 = B ⋅ B =    
1 2 4  2×3 1 2 4  2×3
 −2 −3
=  So B2 is not possible, since the B is not a square
 1 5 matrix.
−1  −2 −3 Hence, (BA)2 ≠ B2 A2 . [3]
A −1 =  
7  1 5
[3]
88 | OSWAAL NCERT SOLUTIONS – Textbook + Exemplar – MATHEMATICS : Class-XII

Q. 29. If possible, find the value of BA and AB, where Q. 31. If X and y are 2 × 2 matrices, then solve the
4 1  following matrix equations for X and Y
2 1 2  2 3  2 3   −2 2 
A=  and B =   2 X + 2Y =  , 3 X + 2Y = 
1 2 4   
1 2  4 0  1 −5 .
.
 [NCERT Exemp. Ex. 3.3, Q. 15, Page 54]  [NCERT Exemp. Ex. 3.3, Q. 19, Page 55]
Ans. We have, Ans. We have,
4 1  2 3
2 1 2 2 3                  2 X + 2Y =   …(i)
A=  and B = 4 0
 
1 2 4  1 2   −2 2 
and 3X + 2Y =   …(ii)
So, AB and BA both are possible.  1 −5
[Since, in both AB and BA, the number of columns On subtracting Eq. (i) from Eq. (ii), we get:
of first is equal to the number of rows of second.]  −2 − 2 2 − 3 
∴ (3X + 2Y ) − ( 2 X + 3Y ) = 
4 1  1 − 4 −5 − 0 
2 1 2  
AB =   2 3   −4 −1
1 2 4  2×3 1 2 
                ( X − Y ) =   …(iii)
3× 2  −3 −5
8 + 2 + 2 2 + 3 + 4  On adding Eqs. (i) and (ii), we get:
= 
4 + 4 + 4 1 + 6 + 8  0 5 
(5 X + 5Y ) =  
12 9  5 −5
= 
12 15 1 0 5 
(X + Y) =  
5 5 −5
4 1 
2 1 2
BA =  2 3   0 1 
1 2 4  2×3 = 
1 2  3×2  1 −1  …(iv)
8 + 1 4 + 2 8+4  On adding Eqs. (iii) and (iv), we get:
=  4 + 3 2 + 6 4 + 12   −4 0 
(X − Y) + (X + Y) =  
 2 + 2 1 + 4 2 + 8   −2 −6 
9 6 12   −2 0 
2X = 2  
= 7 8 16   −1 −3
 4 5 10   −2 0 
[3] ∴ X= 
 −1 −3
2  3   −8 
Q. 30. Solve for x and y, x   + y   +  =0. From Eq. (iv), we have
1  5   −11
 [NCERT Exemp. Ex. 3.3, Q. 18, Page 54]  −2 0  0 1 
 −1 −3 + Y = 1 −1
Ans. We have,    
2 3  −8  2 1   −2 0 
x  + y  +  =0 Y=  and X =  − − 
1
  5  −11  2 2   1 3 [3]
 2 x   3 ⋅ y   −8  Q. 32. If A = [3 5] and B = [7 3] then find a non-zero
⇒  + + =0
 x  5 ⋅ y   −11 matrix C such that AC = BC.
 [NCERT Exemp. Ex. 3.3, Q. 20, Page 55]
 2 x + 3y − 8 0
⇒  x + 5y + 11  = 0 Ans. We have,
    A = [3 5]1× 2 and B = [ 7 3]
∴ 2 x + 3y − 8 = 0 1× 2
x 
⇒ 4 x + 6y = 16 …(i) Let C =   is a non-zero matrix of order 2 × 1.
 y  2 ×1
and x + 5y − 11 = 0
x 
⇒ 4 x + 20y = 44 …(ii) AC = [3 5]  
y
On subtracting Eq. (i) from Eq. (ii), we get:
= [3x + 5 y ]
14 y = 28        
⇒ y=2 x 
and BC = [ 7 3]  
∴ 2x + 3 × 2 − 8 = 0 y
⇒ 2x = 2 = [ 7 x + 3y ]
⇒ x =1    For AC = BC,
∴ x = 1 and y = 2
[3] [3x + 5y ] = [7x + 3y ]
MATRICES | 89
On using equality of matrix, we get 2 3   1 0
3x + 5y = 7x + 3y (ii)      
(B + C ) =  + 
⇒   4x = 2y 3 −4   −1 0 
1 1 3 3 
⇒ x⇒ = x y= y = 
               2 2  2 −4 
⇒              y = 2x  1 2  3 3 
and A ⋅ ( B + C ) =   
 x  x   −2 1   2 −4 
∴C ∴ = C = 
2x 2x
                   =
 3+ 4 3−8 

We see that on taking Column of order 2 × 1, 2 × 2,  −6 + 2 −6 − 4 
2 × 3, …, we get  7 −5
=
∴C =   , 
x  x x  x x x  −4 −10  …(iii)
,  ...            
 2x   2x 2x   2x 2x 2x 
 1 2 2 3 
In general, Also,      AB =   
k  k k   −2 1  3 −4 
∴C =   ,  , etc.
 2k   2k 2k   2+ 6 3−8
=
[3] 
 −4 + 3 −6 − 4 
 1 2 2 3  1 0 
Q. 33. If A =   ,B =   and C =   , verify:  8 − 5
 −2 1  3 −4   −1 0  = 
(i) (AB)C = A(BC).                −1 −10 
(ii) A(B + C) = AB + AC. And
 [NCERT Exemp. Ex. 3.3, Q. 22, Page 55]  1 2  1 0
Ans. We have, AC =   
 −2 1   −1 0 
 1 2 2 3   1 0  1 − 2 0
A=  , B = 3 −4  and C =  −1 0  = 
 −2 1       −2 − 1 0 
 1 2 2 3   −1 0 
(i)      
( AB) =    = 
 −2 1  3 −4   −3 0 
          
2 + 6 3 − 8 
=   8 − 5  −1 0 
 −4 + 3 −6 − 4  ∴ AB + AC =  + 
 −1 −10   −3 0 
 8 −5
=   7 − 5
 −1 −10  ⇒ AB + AC =   …(iv)
 8 − 5   1 0  −4 −10  
and      ( AB)C =   
 −1 −10   −1 0  From Eqs. (iii) and (iv), we have
A(B + C) = AB + AC [3]
8 + 5 0
=  5 3 4   −1 2 1 
 −1 + 10 0  Q. 34. If A A = [ 2 1] , B =   and C =  1 0 2  ,
 8 7 6   
13 0  then verify that A(B + C) = (AB + AC).
=  (i)
                                            9 0  …  [NCERT Exemp. Ex. 3.3, Q. 25, Page 55]
Ans. We have to verify that, A(B + C) = AB + AC
2 3   1 0
Again, ( BC ) =    We have,
3 −4   −1 0  5 3 4   −1 2 1 
A = [ 2 1] , B =   and C =  1 0 2 
2 − 3 0 8 7 6   
= 
3 + 4 0  5 − 1 3 + 2 4 + 1 
 −1 0  ∴ A( B + C ) = [ 2 1]  
=      8 + 1 7 + 0 6 + 2 
7 0 
                                             4 5 5
= [ 2 1]  
 1 2   −1 0  9 7 8 
and     A( BC ) =                                              
 −2 1   7 0  = [8 + 9 10 + 7 10 + 8]
 −1 + 14 0               
=  = [17 17 18]
 +2 + 7 0          …(i)
13 0  5 3 4 
=  …(ii)           Also, AB = [ 2 1]  
9 0 8 7 6 
                                            
∴ (AB)C = A(BC) [By using Eqs. (i) and (ii)] [3] = [10 + 8 6 + 7 8 + 6]
= [18 13 14]
                     
90 | OSWAAL NCERT SOLUTIONS – Textbook + Exemplar – MATHEMATICS : Class-XII

 −1 2 1  Now, P(k) : (A′)k = (Ak)’


         and AC = [ 2 1]   where k ∈ N
 1 0 2 and P(k + 1) : (A′)k + 1 = (Ak + 1)′
= [ −2 + 1 4 + 0 2 + 2] where P(k + 1) is true whenever P (k) is true.
∴ P(k + 1) : (A′)k . (A′)1 = [Ak + 1]′
                                       [
= −1 4 4]
             (Ak)′. (A)′ = [Ak + 1]′
∴ AB + AC = [18 13 14] + [ −1 4 4] (A · Ak)′ = [Ak + 1]′     [∵ (A′)k = (Ak)′ and (AB) = B′A′]
…(ii) (Ak + 1)′ = [Ak + 1]′
           = [17 17 18]  Hence proved. [3]
∴ A( B + C ) = ( AB + AC ) [By using Eqs. (i) and (ii)] Q. 37. Find inverse, by elementary row operations (if
Hence proved. [3] possible), of the following matrices:
1 0 −1   1 3  1 −3 
  (i)  (ii) 
Q. 35. If A =  2 1 3  , then verify that A2 + A = A(A  −5 7 

 −2 6 
0 1 1  [NCERT Exemp. Ex. 3.3, Q. 37, Page 57]
+ I) where I is 3 × 3 unit matrix.  1 3
Ans. Let A =  
 [NCERT Exemp. Ex. 3.3, Q. 26, Page 56]
 −5 7 
Ans. We have, In order to use elementary row operations we may
1 0 −1 write A = IA.
A = 2 1 3  1 3  1 0 
0 1 1 ∴ 
 = A
 −5 7  0 1 
A2 = A ⋅ A
1 3  1 0 
⇒ = A [Q R2 → R2 + 5R1 ]
1 0 −1 1 0 −1
0 22  5 1 
= 2 1 3 2 1 3
0 1 1 0 1 1 1 3  1 0   1 
 ⇒  =  A Q R2 → R2 
1 −1 −2  0 1  5 / 22 1 / 22   22 
= 4 4 4 1 0  7 / 22 −3 / 22 
2
 2 4  ⇒  =  A [Q R1 → R1 − 3R2 ]
0 1  5 / 22 1 / 22 
1 −1 −2  1 0 −1 1 0  1 7 −3
∴ A2 + A = 4 4 4  + 2 1 3 ⇒  =  A
2 2 4  0 1 1  0 1  22 5 1

 2 −1 −3  ⇒ I = BA, where B is the inverse of A.
= 6 5 7  1 7 −3
2 3 B=
 5 …(i) 
22 5 1
 [2]
Now,  1 −3
(ii) Let A =  
1 0 −1 1 0 0 
 −2 6 
A + I =  2 1 3  + 0 1 0  In order to use elementary row operations, we
0 1 1 0 0 1  write A = IA
 1 −3 1 0 
 2 0 −1 ⇒ = A
=  2 2 3   −2 6  0 1 
0 1 2  1 −3 1 0 
         ⇒  = A [Q R2 → R2 + 2R1 ]
0 0   2 1 
and
1 0 −1  2 0 −1  Since, we obtain all zeroes in a raw of the matrix A
on LHS, so A–1 does not exist. [2]
A( A + I ) =  2 1 3  ⋅  2 2 3 
Q. 38. Find the values of x, y, z and w.
0 1 1 0 1 2 
 xy 4  8 W 
 2 −1 −3  If  = .
 z + 6 x + y  0 6 
= 6 5 7   [NCERT Exemp. Ex. 3.3, Q. 38, Page 57]
 2 3 5 (ii)
      … Ans. We have,
Thus, we see that A2 + A = A(A + I) [By using Eqs.  xy 4  8 W 
(i) and (ii)] [3]  z + 6 x + y  = 0 6 
   
Q. 36. Prove by Mathematical Induction that (A′)n = (An)′
where n ∈ N for any square matrix A. By equality of matrix, x + y = 6 and xy = 8
 [NCERT Exemp. Ex. 3.3, Q. 36, Page 57] ⇒ x = 6 – y and (6 – y)y = 8
Ans.        Let P(n) : (A′)n   = (An)′ ⇒        y2 – 4y – 2y + 8 = 0
∴          P(1) : (A)1 = (A)′ ⇒                          (y – 2)(y – 4) = 0
⇒                                        A′   = A′ ⇒ P(1) is true. ⇒                           y = 2 or y = 4
MATRICES | 91
∴           x = 6 – 2  = 4 [By using Eqs. (i) and (iii)]
or x = 6 – 4 = 2 [Because x = 6 − y]  145 −125  42 −70 
  Also, z + 6 = 0 = + 
 −100 120   −56 28 
⇒       z = –6 and W = 4
∴        x = 2, y = 4 or x = 4, y = 2, z = –6 and W  187 −195 
=
        =4 [3]  −156 148
[3]
1 5  9 1  Q. 41. Find the matrix A such that
Q. 39. If A =   and B = 7 8  , find a matrix C
7 12     2 −1  −1 −8 −10 
such that 3A + 5B + 2C is a null matrix.  1 0  A =  1 −2 − 5 
 [NCERT Exemp. Ex. 3.3, Q. 39, Page 57]    
 −3 4   9 22 15
Ans. We have, .
1 5  9 1   [NCERT Exemp. Ex. 3.3, Q. 42, Page 58]
A=  and B = 7 8 Ans. We have,
 7 12   
 2 −1  −1 −8 −10 
 1 0  A =  1 −2 − 5
1 5  9 1  
A= and B =  
7 12  
 7 8  −3 4   9 22 15 3×3
3× 2
a b
Let C =  From the given equation, it is clear that order of A
c d  should be 2 × 3.
∴ 3A + 5B + 2C = 0 a b c 
LetA = 
3 15   45 5   2 a 2b   0 0 d e f 
⇒ + + =
 21 36 35 40  2 c 2d  0 0  2 −1  −1 −8 −10 
 48 + 2 a 20 + 2b  0 0  1  a b c   1 
∴  0  d e f  = −2 − 5
⇒  =
56 + 2 c 76 + 2d  0 0  −3 4     9
 22 15
⇒ 2 a + 48 = 0 ⇒ a = −24  2a − d 2b − e 2c − f   −1 −8 −10 
Also, 20 + 2b = 0 ⇒ b = −10 ⇒  a + 0d  
b + 0⋅e c + 0⋅ f  =  1 −2 − 5
56 + 2 c = 0 ⇒ c = −28  −3a + 4d −3b + 4e −3c + 4 f   9 22 15
and 76 + 2d = 0 ⇒ d = −38  2a − d 2b − e 2c − f   −1 −8 −10 
 −24 −10 
⇒ 
  
a b c  =  1 −2 − 5
∴ c= 
[3]  −28 −38  −3a + 4d −3b + 4e −3c + 4 f   9 22 15
 3 −5 2 By equality of matrices, we get
Q. 40. If A =   then find A − 5A − 14I. Hence,
 −4 2             a = 1, b = –2, c = –5 and 2a – d = –1
⇒                d = 2a + 1 = 3;
obtain A3. [NCERT Exemp. Ex. 3.3, Q. 40, Page 58]
⇒ 2b – e = –8
Ans. We have,
⇒                 e = 2(–2) + 8 = 4
 3 −5            2c – f = –10
A=
−4 2 
…(i)
            ⇒                 f = 2c + 10 = 0
A2 = A ⋅ A 1 −2 −5
A=  [3]
 3 −5  3 −5                        3 4 0 
=
 −4 2   −4 2  Q. 42. If AB = BA for any two square matrices, then
          
prove by mathematical induction that (AB)n =
 29 −25 AnBn.  [NCERT Exemp. Ex. 3.3, Q. 49, Page 58]
=  …(ii)
−20 24  Ans. Let P(n) : (AB)n = AnBn
           
∴      P(1) : (AB)1 = A1B1
 29 −25  15 −25 14 0 
∴ A 2 − 5 A − 14 I =  – –  ⇒                                 AB = AB
 −20 24   −20 10   0 14  So, P(1) is true.
0 0  Now, P(k) : (AB)k = AkBk, k ∈ N
=  So, P(K) is true, whenever P(k + 1) is true.
0 0
                                                       ∴ P(K + 1 : (AB)k + 1 = Ak + 1 Bk + 1
Now,        A2 – 5A – 14I = 0 ⇒ ABk AB1 …(i)
⇒              A·× A2 – 5A·A – 14AI = 0 ⇒ Ak Bk BA ⇒ AkBk + 1 A[∵ AB = BA]
⇒ A3 – 5A2 – 14A = 0 [∵ AI = A] Ak · A·Bk + 1 ⇒ Ak + 1 Bk + 1
⇒              A3 = 5A2 + 14A (AB)k + 1 = Ak + 1 Bk + 1
 29 −25  3 −5 So, P(k + 1) is true for all n ∈ N, whenever P(k) is
= 5  + 14  −4 2 
 − 20 24    true.
92 | OSWAAL NCERT SOLUTIONS – Textbook + Exemplar – MATHEMATICS : Class-XII

By mathematical induction, (AB) = AnBn is true for 2 1 4


all n ∈ N. [3]
A ' = 3 −1 1 

0 2 y z 
1 2 2 
Q. 43. Find x, y and z, if A =  x y − z  satisfies A′=A-1
   4 4 5
 x − y z  Now,
A + A' 1 
=  4 −2 3 
 [NCERT Exemp. Ex. 3.3, Q. 50, Page 59] 2 2
5 3 4 
Ans. We have,
 5
0 2 y z 0 x x 2 2
2

A = x  
y − z  and A ' =  2 y y − y   
3
 x − y z   z =  2 −1
− z z   2
5 3 
Also, A′ = A–1  2
⇒     AA′ = AA–1[∵ AA–1 = I]  2 2 
⇒     AA′ = I  0 2 −3
A − A' 1 
0 2 y z 0 x x  1 0 0  and =  −2 0 1 
2 2
x  
 y −z  2y y − y  = 0 1 0   3 −1 0 
 −3 
 x − y z   z − z z  0 0 1   0 1
2
 4y 2 + z2 2y 2 − z2 −2 y 2 + z 2  1 0 0   
1
    =  −1 0 
 2y − z x + y2 + z2 x 2 − y 2 − z 2  = 0 1 0 
2 2 2
 2 
 x 2 + y 2 + z 2  0 0 1   3 −1 
 −2 y + z x2 − y2 − z2
2 2
 0 
2y 2 − z2 = 0  2 2 
 5  −3 
2y 2 = z2 2 2 0 1
2  2
4y 2 + z2 = 1    
A + A' A − A'  3  1
+ = 2 −1 + −1 0
2 ⋅ z2 + z2 = 1 2 2  2  2
5 3   3 −1 
1
z=±  2  0
3  2 2   2 2 
z2 This is the required expression. [3]
y =
2

2 Q. 45. Find a matrix D such that CD – AB = 0. Let


1  2 −1  5 2 2 5
y=± A= ,B =   and C =  .
6 3 4 7 4 3 8
Also, x2 + y2 + z2 = 1
 [CBSE Board, Delhi Region, 2017]
x2 = 1 − y2 − z2 Ans. Let,
1 1 x y 
=1− − D=
z w 
6 3 [½]
3               
=1− CD = AB
6
1  2 x + 5 z 2 y + 5w   3 0 
= ⇒  = 
2 3x + 8z 3y + 8w   43 22  [1+1]
1
x=± 2x + 5z = 3, 3x + 8z = 43; 2y + 5w = 0, 3y + 8w = 22
2 Solving we get x = –191, y = –110, z = 77, W = 44 [1]
1 1 1  −191 −110 
Therefore, x = ± , ,y = ± and z = ± [3] ∴D = 
2 6 3  77 44 
[½]
2 3 1  2 −1   −1 − 8 
Q. 44. Express the matrix 1 −1 2  as the sum of a   
Q. 46. Find matrix A such that  1 0  A =  1 −2  .

 
 4 1 2   −3 4  
  9 22 
symmetric and a skew-symmetric matrix.  [CBSE Board, All India Region, 2017]
 [NCERT Exemp. Ex. 3.3, Q. 52, Page 59] a b 
Ans. We have, Ans. Let A =  
c d
2 3 1
   2 −1   −1 −8 
A = 1 −1 2    a b   
 1 0    =  1 −2 
 4 1 2   −3 4   c d   9 22 
   
[1]
MATRICES | 93

 2a − c 2b − d   −1 −8  ∴A⋅X = B
   
⇒ a b  =  1 −2  1  5 −6 
 −3a + 4 c −3b + 4d   9 22  A = −11; A −1 =  
    −11  −6 5 
[1]                [1]
⇒          2a – c = –1, 2b – d = –8 ∴ Solution is 
       a = 1, b = –2 [1] 
X = A −1B 
               –3a + 4c  = 9, –3b + 4d = 22
x  1  5 −6  140000  
Solving to get a = 1, b = –2, c = 3 and d = 4  =    
1 −2   y  −11  −6 5  135000  
∴A =  
 10000  
3 4  [1] = 
15000 
  
 1 2 3   −7 −8 −9 
Q. 47. Find matrix X so that X  = . ∴ x = 10, 000 and y = 15, 000 
4 5 6  2 4 6  
∴ Amount invested = Rs. 25, 000 
 [CBSE Board, Foreign Scheme, 2017] [½+½]
a b  Value : Caring elders [1]
Ans. Let            X =   [1] Q. 49. A coaching institute of English (subject) conducts
c d
classes in two batches I and II and fees for rich and
 a b 1 2 3   −7 −8 −9  poor children are different. In batch I, it has 20 poor
then,   = 
       c d  4 5 6   2 4 6 and 5 rich children and total monthly collection is
`9,000, whereas in batch II, it has 5 poor and 25 rich
 a + 4b 2a + 5b 3a + 6b   −7 −8 −9 
⇒  =  children and total monthly collection is `26,000.
c + 4d 2c + 5d 3c + 6d   2 4 6
   [1] Using matrix method, find monthly fees paid by
Equating and solving to get a = 1, b = –2, c = 2 and each child of two types. What values the coaching
d=0 [1½] institute is inculcating in the society?
 [CBSE Board, Foreign Scheme, 2016]
1 −2 
X =  Ans. Let each poor child Pay `x per month and each rich
2 0 [½] child pay `y per month.
Q. 48. A trust invested some money in two types ∴ 20x + 5y = 9000 [½]
of bonds. The first bond pays 10% interest   5x + 25y = 26000 [½]
and second bond pays 12% interest. The trust In matrix form, we have
received `2,800 as interest. However, if trust had  20 5   x  9000 
interchanged money in bonds, they would have    =  
got `100 less as interest. Using matrix method,  5 25  y   26000  [1]
find the amount invested by the trust. Interest AX = B
received on this amount will be given to Helpage ⇒ X = A −1B
India as donation. Which value is reflected in this ⇒     
question?  [CBSE board, All India Region, 2016] 1  25 −5
A −1 = 
Ans. Let `x be invested in first bond and `y be invested 475  −5 20 
           
in second bond then the system of equations is:
x  1  25 −5 9000 
10x 12 y   = 
+ = 2800 
 5x + 6 y = 140000   y  475  −5 20   26000 
100 100
⇒   200 
12x 10 y 6x + 5 y = 135000 
+ = 2700  = 
100 100  1000 
[1]
5 6  x  140000  ⇒ x = 200, y = 1000
Let     A =   ; X =  y  ; B = 135000  ⇒     [1]
 6 5      Value : Compassion or any relevant value [1]

Long Answer Type Questions (5 or 6 marks each)

Q. 1. Construct a 3 × 4 matrix, whose elements are given 1 1 1 2


by:
∴a11 = −3 × 1 + 1 = −3 + 1 = −2 = = 1
2 2 2 2
1 1 1 1 5
(i) aij = −3i + j (ii) aij = 2 i − j a21 = −3 × 2 + 1 = −6 + 1 = −5 =
2 2 2 2 2
 [NCERT Ex. 3.1, Q. 5, Page 64]
1 1 1 8
Ans. In general, a 3 × 4 matrix is given by a31 = −3 × 3 + 1 = −9 + 1 = −8 = = 4
1 2 2 2 2
(i) aij = −3i + j , i = 1, 2, 3 and j = 1, 2, 3, 4 1 1 1 1
2 a12 = −3 × 1 + 2 = −3 + 2 = −1 =
2 2 2 2
1 1 1 4
a22 = −3 × 2 + 2 = − 6 + 2 = − 4 = = 2
2 2 2 2
1 1 1 7
a32 = −3 × 3 + 2 = − 9 + 2 = − 7 =
2 2 2 2
1 1 1 5
a21 = −3 × 2 + 1 = −6 + 1 = −5 =
2 2 2 2
1 1 1 8
a31 = −3 × 3 + 1 = −9 + 1 = −8 = = 4
2 2 2 2
94 | OSWAAL1NCERT SOLUTIONS 1 1
– Textbook + 1Exemplar – MATHEMATICS : Class-XII
a12 = −3 × 1 + 2 = −3 + 2 = −1 =
2 2 2 2
1 1 1 4  2 4   1 3
a22 = −3 × 2 + 2 = −6 + 2 = −4 = = 2 (ii) A − B =  −  [1]
2 2 2 2 3 2   −2 5
1 1 1 7  2 −1 4 − 3
a32 = −3 × 3 + 2 = −9 + 2 = −7 = =
2 2 2 2 
3 − ( −2) 2 − 5
1 1
a13 = −3 × 1 + 3 = −3 + 3 = 0 1 1
2 2 = 
1 1 1 3  5 − 3 
a23 = −3 × 2 + 3 = −6 + 3 = −3 =
2 2 2 2  2 4   −2 5 
1 1 1 6 (iii) 3 A − C = 3  −  [1]
a33 = −3 × 3 + 3 = −9 + 3 = −6 = = 3 3 2   3 4 
2 2 2 2
3 × 2 3 × 4   −2 5 
1 1 1 1 = − 
a14 = −3 × 1 + 4 = −3 + 4 = 1 = 3 × 3 3 × 2   3 4 
2 2 2 2
6 12   −2 5 
1 1 1 2 = − 
a24 = −3 × 2 + 4 = −6 + 4 = −2 = = 1
2 2 2 2 9 6   3 4 
1 1 1 5 6 + 2 12 − 5
a34 = −3 × 3 + 4 = −9 + 4 = −5 = = 
2 2 2 2 9 − 3 6 − 4 
Therefore, the required matrix is: 8 7 
= 
 1 1 6 2 
1 2 0 2 
  (iv) Matrix A has 2 columns. This number is equal to
5 3
A= 2 1 the number of rows in matrix B. Therefore, AB is
2 2  defined as:
 
4 7 3 5   2 4   1 3
 2 2  AB =   
[2½] 3 2   −2 5
(ii)            aij = 2i − j , i = 1, 2, 3 and j = 1, 2, 3, 4  2(1) + 4( −2) 2(3) + 4(5)
∴ a11 = 2 × 1 – 1 = 2 – 1 = 1 = 
3(1) + 2( −2) 3(3) + 2(5) 
         a21 = 2 × 2 – 1 = 4 – 1 = 3
 2 − 8 6 + 20  −6 23
         a31 = 2 × 3 – 1 = 6 – 1 = 5 = = 
         a12 = 2 × 1 – 2 = 2 – 2 = 0 3 − 4 9 + 10   −1 19  [1]
         a22 = 2 × 2 – 2 = 4 – 2 = 2 (v) Matrix B has 2 columns. This number is equal to
         a32 = 2 × 3 – 2 = 6 – 2 = 4 the number of rows in matrix A. Therefore, BA is
         a13 = 2 × 1 – 3 = 2 – 3 = –1 defined as:
         a23 = 2 × 2 – 3 = 4 – 3 = 1  1 3  2 4 
         a33 = 2 × 3 – 3 = 6 – 3 = 3 BA =   
         a14 = 2 × 1 – 4 = 2 – 4 = –2  −2 5 3 2 
 1( 2) + 3(3) 1( 4) + 3( 2) 
         a24 = 2 × 2 – 4 = 4 – 4 = 0 = 
         a34 = 2 × 3 – 4 = 6 – 4 = 2  − 2 ( 2 ) + 5 ( 3) − 2 ( 4) + 5( 2)
Therefore, the required matrix is:  2+9 4+6 
=
1 0 −1 −2   − 4 + 15 − 8 + 10 
A = 3 2 1 0  =
11 10 

5 4 3 2  11 2  [1]
[2½]
2 4   1 3  −2 5  1 2 −3  3 −1 2  4 1 2
Q. 2. Let A =   ,B =   and C =   . Find A = 5 0 2  , B =  4 2 5  0 3 2 
Q. 3. If and C =
3 2   −2 5   3 4      
each of the following: 1 −1 1   2 0 3  1 −2 3 
(i) A + B (ii) A – B then compute (A + B) and (B – C). Also, verify that
(iii) 3A – C (iv) AB A + (B – C) = (A + B) – C.
(v) BA [NCERT Ex. 3.2, Q. 1, Page 80]  [NCERT Ex. 3.2, Q. 4, Page 81]
Ans. 1 2 -3  3 −1 2 
(i) A + B = 
 2 4   1 3
+ [1] A + B = 5 0 2  +  4 2 5 
 
3 2   −2 5 1 −1 1   2 0 3 
 2 + 1 4 + 3 1 + 3 2 − 1 −3 + 2 
=   + 
 3 − 2 2 + 5  =  5 4 0 +2 2 + 5
3 7  1 + 2 −1 + 0 1 + 3 
=   4 1 −1
1 7 
Ans. = 9 2 7  [2]

3 −1 4 
= 
 2 8
1 10 0 
X=  
2  2 8
5 0  MATRICES | 95
= 
1 4 
3 −1 2   4 1 2 Now,
= B − C  4 2 5  − 0 3 2  X +Y =
7 0 
2 5
 2 0 3  1 −2 3   
3 − 4 − 1 − 1 2 − 2 5 0  7 0 
 −  1 4  + Y =2 5
=4 0 2−3 5 − 2    
 2 − 1 0 − ( −2) 3 − 3   7 0  5 0 
= Y  − 
 −1 −2 0   2 5  1 4 
=  4 −1 3  7 − 5 0 − 0
= 
       1 2 0  [2] 2 − 1 5 − 4
1 2 3   −1 −2 0  2 0
= 
A +=( B − C ) 5 0 2  +  4 −1 3  1 1 
1 1 1   1 2 0  2 3
2 X + 3Y =4 0 …(iii)
1 + ( −1) 2 + ( −2) −3 + 0   
(ii)       
= 5 + 4 0 + ( −1) 2 + 3 
 2 −2 
1 + 1 −1 + 2 1 + 0  3X + 2Y =
 −1 …(iv)
          5 
 0 0 3 
Multiplying equations (iii) with (2), we get:
= 9 −1 5 
2 3
 2 1 1  2( 2 X + 3Y ) =
2 
4 0
 4 1 1  4 1 2 
4 6
+ B) − C 9 2 7  − 0 3 2 
( A= ⇒ 4 X + 6Y =8 0  …(v)
3 −1 4  1 −2 3    
Multiplying equations (iv) with (3), we get:
4 − 4 1 − 1 −1 − 2 
   2 −2 
=−  9 0 2 − 3 7 − 2 3(3X + 2Y ) =3 
 −1 5 
3 − 1 −1 − −2 4 − 3 
 0 0 3  6 −6 
⇒ 9 X + 6Y =  −3 15 …(vi)
= 9 −1 5    
 2 1 1 From equations (v) and (vi), we have:
Hence, we have verified that A + (B – C) =  4 6   6 −6 
( 4 X + 6Y ) − (9 X + 6Y=)  − 
(A + B) – C. [3] 8 0   −3 15
Q. 4. Find X and Y, if  4−6 6 − ( −6)
−5 X =  
7 0  3 0  8 − ( −3) 0 − 15 
(i) X + Y 
=  =
and X −Y  
2 5  0 3   −2 12 
= 
2 3   2 −2   11 −15 
(ii)=2 X + 3Y   =
and 3 X + 2Y   1  −2 12 
4 0  −1 5  X= −  
 [NCERT Ex. 3.2, Q. 7, Page 81] 5  11 −15
Ans.  2 12 
 5 −5
7 0  = 
(i) X + Y =   …(i)  − 11 3
2 5
 5  [2½]
3 0 
X −Y = 0 3  2 3
  …(ii) Q 2 X + 3Y =
4 0
 
Adding equations (i) and (ii), we get:
 7 0  3 0   2 12 
=2X  +   5 −5 2 3
 2 5  0 3  2  + 3Y = 4 0
− 11
7 + 3 0 + 0  3  
=   5 
2 + 0 5 + 3   4 24 
10 0   5 − 5 2 3
=    + 3Y =
 4 0
 2 8  − 22 6  
 5 
1 10 0 
X=    4 24 
2  2 8 − 
2 3  5 5
5 0  =
3Y  − 
=   4 0   − 22 6
1 4   5 
Now,  4 24 
2 − 5 3+ 
7 0  3Y =  5

X +Y =2 5 22
  4 + 0−6 
 5 
  + 3Y =  
 − 22 6  4 0
 5 
 4 24 
− 
2 3  5 5
3Y = − 
96 | OSWAAL NCERT SOLUTIONS4 0   22 – Textbook + Exemplar – MATHEMATICS : Class-XII
− 6
 5 
 4 24   −1 1 0  1 2 3 
2 − 5 3+ 
5  0 −1 1  0 1 0 
3Y =     
 4 + 22 0 − 6   2 3 4  1 1 0 
 5 
6 39   −1(1) + 1( 0) + 0(1) −1( 2) + 1(1) + 0(1) 
5   + − + 
5  0(1) ( 1 )( 0 ) 1(1) 0( 2) + ( −1)(1) + 1(1)
= 
 42 −6   2(1) + 3( 0) + 4(1) 2( 2) + 3(1) + 4(1) 
 5  = 
 −1(3) + 1( 0) + 0( 0) 
6 39  0(3) + ( −1)( 0) + 1( 0) 
1 5 5   
Y=    2(3) + 3( 0) + 4( 0) 
3  42
−6   −1 −1 −3
 5 
 2 13  =  1 0 0 
 5  6 11 6 
= 5 
14 −2  1 2 3   −1 1 0   −1 1 0  1 2 3 
 5  [2½]
∴ 0 1 0   0 −1 1  ≠  0 −1 1  0 1 0 
Q. 5. Show that
5 −1   2 1   2 1  5 −1 1 1 0   2 3 4   2 3 4  1 1 0   [2½]
(i)     ≠   
6 7  3 4  3 4  6 7  2 0 1 
2  
1 2 3   −1 1 0   −1 1 0  1 2 3  Q. 6. Find A – 5A + 6I, if A =  2 1 3 
(ii) 0 1 0   0 −1 1  ≠  0 −1 1  0 1 0  1 −1 0 
     
1 1 0   2 3 4   2 3 4  1 1 0   [NCERT Ex. 3.2, Q. 15, Page 82]
Ans. We have A2 = A × A
 [NCERT Ex. 3.2, Q. 14, Page 82]
Ans. 2 0 1  2 0 1 
A = AA =  2 1 3  2
2
1 3
(i) 5 −1  2 1  5( 2) − 1(3) 5(1) − 1( 4) 
    =  1 −1 0 1 −1 0
6 7  3 4  6( 2) + 7(3) 6(1) + 7( 4)
10 − 3 5 − 4   2( 2) + 0( 2) + 1(1) 2(0) + 0(1) + 1( −1) 
=   2( 2) + 1( 2) + 3(1) 
12 + 21 6 + 28  2(0) + 1(1) + 3( −1) 
 7 1 1( 2) + ( −1)( 2) + 0(1) 1(0) + ( −1)(1) + 0( −1) 
= = 

33 34   2(1) + 0(3) + 1(0) 
 2(1) + 1(3) + 3(0) 
 2 1  5 −1  2(5) + 1( 6) 2( −1) + 1( 7 )   
  =  1(1) + ( −1)(3) + 0(0) 
3 4  6 7  3(5) + 4( 6) 3( −1) + 4( 7 )
10 + 6 −2 + 7   4 + 0 + 1 0 + 0 − 1 2 + 0 + 0
=
15 + 24 −3 + 28
 =  4 + 2 + 3 0 + 1 − 3 2 + 3 + 0 
16 5   2 − 2 + 0 0 − 1 + 0 1 − 3 + 0 
= 
39 25 5 −1 2 
Therefore, = 9 −2 5 
5 −1  2 1   2 1  5 −1  0 −1 −2 
  ≠  
 6 7  3 4  3 4   6 7 [2½] Therefore,
(ii) 5 −1 2  2 0 1  1 0 0
1 2 3   −1 1 0  A 2 − 5 A + 6I = 9 −2 5  − 5  2 1 3  + 6 0 1 0 
  
0 1 0   0 −1 1  0 −1 −2  1 −1 0  0 0 1 
1 1 0   2 3 4 
 5 −1 2   10 0 5  6 0 0
1( −1) + 2( 0) + 3( 2) 1(1) + 2( −1) + 3(3)   
= 9 −2 5  − 10  5 15 + 0 6 0 
0( −1) + 1( 0) + 0( 2) 0(1) + 1( −1) + 0(3)
  0 −1 −2   5 −5 0  0 0 6 
1( −1) + 1( 0) + 0( 2) 1(1) + 1( −1) + 0(3) 
=  5 − 10 −1 − 0 2 − 5  6 0 0 
1( 0) + 2(1) + 3( 4)  = 9 − 10 −2 − 5 5 − 15  + 0 6 0 
0( 0) + 1(1) + 0( 4) 
  0 − 5 −1 + 5 −2 − 0  0 0 6 
1( 0) + 1(1) + 0( 4) 
 −5 −1 − 3  6 0 0 
 5 8 14  =  −1 −7 −10  + 0 6 0 
=  0 −1 1 
 −5 4 − 2  0 0 6 
 −1 0 1   −5 + 6 −1 + 0 −3 + 0 
 
=  −1 + 0 −7 + 6 −10 + 0 
 −5 + 0 4 + 0 −2 + 6 
   
= 9 − 10 −2 − 5 5 − 15  + 0 6 0 
0 − 5 −1 + 5 −2 − 0  0 0 6 
 −5 −1 − 3  6 0 0 
=  −1 −7 −10  + 0 6 0  MATRICES | 97
 −5 4 − 2  0 0 6 
Ans. On the LHS
 −5 + 6 −1 + 0 −3 + 0 
I+A
=  −1 + 0 −7 + 6 −10 + 0 
 a
 −5 + 0 4+0 −2 + 6  0 − tan 
1 0  2
=  +  
 1 −1 − 3
0 1   tan a
  0 
=  −1 −1 −10   2 
 −5 4 4 
[5]  a
 1 − tan 
1 0 2  = 2

   tan a 
Q. 7. If A = 0 2 1  prove that A3 – 6A2 + 7A + 2I = 0. 1 …(i)
 2 0 3   2  
 [NCERT Ex. 3.2, Q. 16, Page 82] On the RHS
Ans. A 2 = AA
cos α − sin α 
( I − A)  
1 0 2  1 0 2  sin α cos α 
= 0 2 1  0 2 1    α
 1 0  0 − tan  
 2 0 3   2 0 3 
=  − 2  cos α − sin α 
   
1 + 0 + 4 0 + 0 + 0 2 + 0 + 6   0 1   α   sin α cos α 
 tan 0
  
= 0 + 0 + 2 0 + 4 + 0 0 + 2 + 3   2
 2 + 0 + 6 0 + 0 + 0 4 + 0 + 9   α
 1 tan 
= 2 cos α − sin α 

5 0 8 
 − tan α sin α cos α 
=  2 4 5  1 
 2 
8 0 13
 α α
Now, A3 = A2 A cos α + sin α tan 2 − sin α + cos α tan 
2
= 
5 0 8  1 0 2   − cos α tan α + sin α sin α tan α + cos α 
=  2 4 5  0 2 1   2 2 
…(ii)
8 0 13  2 0 3 
5 + 0 + 16 0 + 0 + 0 10 + 0 + 24 
1 − 2 sin


2
a

2
+ 2 sin
a

2
cos
a

2
tan
a

2
−2 sin
a

2
cos
a

2
(
+ 2 cos
2
a

2
)
− 1 tan
a

2


=  2 + 0 + 10 0 + 8 + 0 4 + 4 + 15  =
 
8 + 0 + 26 0 + 0 + 0 16 + 0 + 39 
 21 0 34 

 (
 − 2 cos
2
a

2
)
− 1 tan
a

2
+ 2 sin
a

2
cos
a

2
2 sin
a

2
cos
a

2
tan
a

2
+ 1 − 2 sin
2
a

2



= 12 8 23 1 − 2 sin a + 2 sin a
2 2
−2 sin
a
cos
a
+ 2 sin
a
cos
a
− tan
a 
 2
34 0 55 
2 2 2 2 2 2
=  
∴ A3 – 6A2 + 7A + 2I  −2 sin a cos a + tan a + 2 sin
a
cos
a
2 sin
2
a
+ 1 − 2 sin
2
a

 2 2 2 2 2 2 2 
 21 0 34  5 0 8  1 0 2  1 0 0 
1  a
= 12 8 23 − 6  2 4 5  + 7 0 2 1  + 2 0 1 0 
− tan
34 2
 0 55  8 0 13  2 0 3   2 0 1  =  
 tan a 1 
 21 0 34  30 0 48 7 0 14   2 0 0   2 
= 12 8 23 − 12 24 30  + 0 14 7  + 0 2 0 
34 Thus, from equations (i) and (ii), we get LHS =
 0 55   48 0 78  14 0 21 0 0 2 
RHS [5]
 21 + 7 + 2 0 + 0 + 0 34 + 14 + 0  30 0 48 Q. 9. A trust fund has `30,000 that must be invested in
= 12 + 0 + 0 8 + 14 + 2 23 + 7 + 0  − 12 24 30  two different types of bonds. The first bond pays
5% interest per year, and the second bond pays
34 + 14 + 0 0 + 0 + 0 55 + 21 + 2   48 0 78  7% interest per year. Using matrix multiplication,
0 0 0  determine how to divide `30,000 among the two
  types of bonds. If the trust fund must obtain an
= 0 0 0 
0 0 0  annual total interest of:
(a) `1800 (b) `2000
=0
 [NCERT Ex. 3.2, Q. 19, Page 82]
∴ A3 – 6A2 + 7A + 2I = 0 [5]
Ans. (a) Let `x be invested in the first bond. Then, the
 a
0 − tan  sum of money invested in the second bond will be
 2
Q. 8. If A =   and I is the identity matrix `(30000 – x).
 tan a 0  It is given that the first bond pays 5% interest per
 2 
cosa − sina  year and the second bond pays 7% interest per year.
of order 2, show that I + A = ( I − A)   Therefore, in order to obtain an annual total
 sina cosa 
 [NCERT Ex. 3.2, Q. 18, Page 82] interest of `1800, we have:
LHS = ( A + B )′
 −5 3 −2  '
=  6 9 9 
98 | 
OSWAAL NCERT SOLUTIONS – Textbook + Exemplar – MATHEMATICS : Class-XII
 −1 4 2 
 5 
 SI for 1 year   −5 6 −1
100 
 
[ x (30000 − x )]  7  =1800  Principal × Rate × Time  =  3 9 4 
  =  −2 9 2 
100   100 

5x 7(30000 − x ) RHS = A′ + B′
⇒ + = 1800
100 100  −1 2 3 '  −4 1 −5 '
⇒ 5x + 210000 − 7 x = 180000    
= 5 7 9 +  1 2 0
⇒ 210000 − 2x = 180000  −2 1 1  1 3 1
⇒ 2x = 210000 − 180000  −1 5 −2   −4 1 1
⇒ 2x = 30000 =  2 7
 
1 +  1 2 3

⇒ x = 15000  3 9 1  −5 0 1
Thus, in order to obtain an annual total interest of
 −1 − 4 5 + 1 −2 + 1
` 1800, the trust fund should invest ` 15000 in the
first bond and the remaining ` 15000 in the second =  2 + 1 7 + 2 1 + 3
bond. [2½]  3 − 5 9 + 0 1 + 1
(b) Let ` x be invested in the first bond. Then, the  −5 6 −1
sum of money invested in the second bond will be  
` (30000 – x). Therefore, in order to obtain an =  3 9 4
annual total interest of ` 2000, we have:  −2 9 2 
 5  ∴ LHS = RHS
  Hence proved. [2½]
[ x (30000 − x )] 100  = 2000
 7   −1 2 3  −4 1 −5
100     
(ii) A − B =  5 7 9  −  1 2 0 
5x 7(30000 − x )  −2 1 1  1 3 1
⇒ + = 2000
100 100  −1 + 4 2 − 1 3 + 5
⇒ 5x + 210000 − 7 x = 200000 =  5 − 1 7 − 2 9 − 0 
⇒ 210000 − 2x = 200000  −2 − 1 1 − 3 1 − 1
⇒ 2x = 210000 − 200000  3 1 8
⇒ 2x = 10000 =  4 5 9 
 −3 −2 0 
⇒ x = 5000
Thus, in order to obtain an annual total interest of LHS = ( A − B )′
` 2000, the trust fund should invest ` 5000 in the first
bond and the remaining ` 25000 in the second bond.  3 1 8 '
[2½] =  4 5 9 
 −1 2 3   −4 1 −5  −3 −2 0
   
Q. 10. If A =  5 7 9  and B =  1 2 0  then verify that: 3 4 −3
 −2 1 1   1 3 1  =  1 5 −2 
(i) (A + B)′ = A′ + B′ (ii) (A – B)′ = A′ – B′ 8 9 0
 [NCERT Ex. 3.3, Q. 2, Page 88] RHS = A′ − B′
 −1 2 3  −4 1 −5  −1 2 3 '  −4 1 −5 '
Ans. (i) A + B =  5 7 9  +  1 2 0  =  5 7 9  −  1 2 0 
 −2 1 1  1 3 1  3 9 1  1 3 1
 −1 − 4 2 + 1 3 − 5  −1 5 −2   −4 1 1
=  5 + 1 7 + 2 9 + 0  
= 2 7 1 −  1 2 3
 −2 + 1 1 + 3 1 + 1  3 9 1  −5 0 1
 −5 3 −2   −1 + 4 5 − 1 −2 − 1
=  6 9 9  =  2 − 1 7 − 2 1 − 3
 −1 4 2   3 + 5 9 − 0 1 − 1
3 4 −3
LHS = ( A + B )′
=  1 5 −2 
 −5 3 −2  '
8 9 0 
=  6 9 9 
∴ LHS = RHS
 −1 4 2 
Hence proved. [2½]
 −5 6 −1
=  3 9 4 
 −2 9 2 
MATRICES | 99

 3 4  4 3
 −1 2 1
Q. 11. If A′ =  −1 2  and B =   , then verify that: =  −3 0 
   
 0 1  1 2 3  −1 −2 
(i) (A + B)′ = A′ + B′
RHS = A′ − B′
(ii)    (A – B)′ = A′ – B′ [NCERT Ex. 3.3, Q. 3, Page 80]
Ans. Given that,  3 4
 −1 2 1 '
=  −1 2  − 
 3 4
 −1 2 1  1 2 3
          A′ =  −1 2  and B =   , then  0 1
 0 1  1 2 3  3 4   −1 1
=  −1 2  −  2 2 
( A′)′ = A
 0 1  1 3
 3 −1 0 
=   3 + 1 4 − 1
 4 2 1
=  −1 − 2 2 − 2 
 3 −1 0  −1 2 1
(i) A + B =  +   0 − 1 1 − 3
 4 2 1  1 2 3  4 3
=
 3 − 1 −1 + 2 0 + 1 =  −3 0


 4 + 1 2 + 2 1 + 3  −1 −2 
 2 1 1 ∴ LHS = RHS
= 
 5 4 4 Hence proved. [2½]
LHS = ( A + B )′ Q. 12. Find the inverse of each of the matrices, if it exists
2 1 1 '  2 −3 3 
=
4 4  in given matrices  2 
2 3 .
5
2 5  3 −2 2 
=  1 4   [NCERT Ex. 3.4, Q. 15, Page 97]
 1 4   2 −3 3
RHS = A′ + B′  
Ans. Let A =  2 2 3
 3 4
 −1 2 1 '  3 −2 2 
=  −1 2  + 
1 2 3 We know that A = IA
 0 1 
 2 2 −
−33 3  1
3 1 0
0 00
 3 4   −1 1  2  
∴  = 0  0 1 0 A
=  −1 2  +  2 2  ∴ 2 3
3 =
 2 2  1 0 A
'

 0 1  1 3  33 −2


−2 2  0
2   0 1
0 0 1
 3 − 1 4 + 1  22 −3
− 3 3  1
3 1 00 00
=  −1 + 2 2 + 2  ⇒
⇒  0 5 0   =  −1 1 0 A R → R − R
0 =  −1 1 0 A (( R22 → R22 − R11 ))
 0 5
 0 + 1 1 + 3  3
3 −2
−2 2   0
2 0 00 11
 1 1 00 0 0
 2 5  2 2 −
−33 3 
3 
 0 1 1  1
=  1 4  ⇒
⇒  0 1
1 0 =
0 =  − −1 1 0 0 A A  R → 1 
R2 →
  5 5 5 5    2 5 
5
 3 − 2 2   0 0 1
2
 1 4  3 −2
 0 0 1
∴ LHS = RHS  1 0 −1
Hence proved. [2½]  − −1
1 −
−1
1 1  1 0 −1
1
 0 1   − 1 1 1 1 
A (R R3 ))
⇒ 0 = 0 A R1 → R1 −
 ( 1
⇒ 0 = − 0 →R
 3 −1 0   −1 2 1  0 1  5 5 5 5 1 − R3
(ii) A − B =  −   3 −2 2 
2   0 0 1
 4 2 1  1 2 3  3 −2  0 0 1
 3 + 1 −1 − 2 0 − 1
=   4 4 1 1 
 4 − 1 2 − 2 1 − 3  5 5 − −1 1
 4 −3 −1  −
−11 0
0 1 
1 5 5 
 0  =  − 1 1 1 1 0 A  R R1 →
→R R1 + R2 and
+R and 
=  ⇒ 1 0
⇒  0 1 0 =  − 0 A  R1 → R1 + 2 R 2

 3 0 −2   5 5 5 5   R3 → R33 + 2R22 
 3
 3
0
0 2   2 2
2   3
LHS = (A – B)’  − 2 2 1
− 1
 4 −3 −1 '  5 5 5 5 
=    44 11 
 3 0 –2   5 5 − −11
 −−11 00 11  5 5 
 0  =  − 11 11 0 A R → R + 3R

⇒  0 11 0
0 =  − 0 A (( R33 → R33 + 3R11 ))
 55 55 
 00 00 55  2 1 −2 
 2 1 − 2 
 
 4 1 
 5 5 −1
 −1 0 1  
1 1  R → R1 + R2 and 
⇒  0 1 0 =  − 0 A  1 
 5 5   R3 → R3 + 2 R2 
 3 0 2   
100 | OSWAAL NCERT SOLUTIONS


2 2
1
– Textbook + Exemplar – MATHEMATICS : Class-XII
 5 5 
 4 1  Applying R3 → R3 + R2, we have:
 5 5 −1  1 0 10  −5 0 3
 −1 0 1   0 1 21 =  −13 1 8 A
1 1
⇒  0 1 0 =  − 0 A ( R3 → R3 + 3R1 )    
 5 5  0 0 25  −15 1 9 

 0 0 5 
2 1 −2 
  1
  Applying R3 → R3 , we have:
25
 4 1 
 5 5 −1  
 −1 0 1    1 0 10   −5 0 3
⇒  0 1 0 =  −
1 1 
0 A  R3 → R3 
1     
 5 5   5  0 1 21 =  −13 1 8 A
 0 0 1  2 1 2  0 0 1  3 1 9
 − 
−   5 25 25 
 5 5 5 
 2 3 Applying R1 → R1 –10R3, and R2 → R2 – 21R3, we
 5 0 − 5 have:
 −1 0 0  
1 1  2 3
⇒  0 1 0 =  − 0 A ( R1 → R1 − R3 )  1 − 5 − 5
 5 5   1 0 0  
 0 0 1 
2 1 2 0 1 0  =  − 2 4 11 
 −     5 25 25  A
 5 5 5  0 0 1  
 2 3 − 3 1 9
− 0 5   5 25 25 
 1 0 0  5 
   1 1  2 3
0 1 0 =  − 5 5 0 A ( R1 → ( −1) R1 )
⇒   1 − 5 − 5
0 0 1   
2 1 2 ∴ A −1 =  −
2 4 11 
 −   5 25 25 
 5 5 5   
− 3 1 9
 2 3  5 25 25 
− 5 0  [5]
5 Q. 14. Find the inverse of each of the matrices, if it exists
 
1 1
∴ A −1 =  − 0  2 0 −1
 5 5 
  in given matrices  5 1 0  .
 
 2 1 − 2  0 1 3 
 5 5 5
[5]  [NCERT Ex. 3.4, Q. 17, Page 97]
Q. 13. Find the inverse of each of the matrices, if it exists
 2 0 −1
 1 3 −2 
Ans. Let A =  5 1 0 
in given matrices  −3 0 −5 .
 0 1 3
 2 5 0 
We know that A = IA
 [NCERT Ex. 3.4, Q. 16, Page 97]
 2 0 −1  1 0 0 
 1 3 −2 
  ∴  5 1 0  = 0 1 0  A
Ans. Let A =  −3 0 −5  0 1 3 0 0 1
 2 5 0 
We know that A = IA Applying R1 → 1 R1 , we have:
 1 3 −2   1 0 0  2

∴  −3 0 −5 = 0 1 0  A     1   1 
 1 0 − 2   2 0 0
 2 5 0  0 0 1    
5 1 0 =  0 1 0 A
Applying R2 → R2 + 3R1 and R3 → R3 – 2R1, we 0 1 3  0 0 1
have:    
   
 1 3 − 2   1 0 0 
0 9 −11 =  3 1 0  A Applying R 2 → R 2 – 5R 1, we have:
     1   1 
0 −1 4   −2 0 1 1 0 − 2   2 0 0
   
Applying R1 → R1 + 3R3 and R2 → R2 + 8R3, we 0 1 5  5
= − 1 0 A
have:  2  2 
   
 1 0 10   − 5 0 3   0 1 3   0 0 1 
       
 0 1 21  = − 13 1 8  A
0 −1 4   −2 0 1 Applying R → R – R , we have:
3 3 2
MATRICES | 101

 1  1   1 1 1
1 0 − 2  2 0 0
Q. 16. If A = 1 1 1 Prove that   
   
0 1 5  5
= − 1 0 A 1 1 1
 2  2 
    3n − 1 3n − 1 3n − 1 
0 0 1  5
−1 1  
 2   2  A = 3n − 1 3n − 1 3n − 1  , n ∈ N
n

3 n − 1
3 n − 1
3 
n − 1
Applying R3 → 2R3, we have:       .
 1  1   [NCERT Misc. Ex. Q. 2, Page 100]
1 0 − 2   2 0 0
Ans. It is given that,
   
0 1 5  5
= − 1 0 A 1 1 1
 2  2 
    A = 1 1 1
0 0 1  5 −2 2 
1 1 1
   
3n −1 3n −1 3n −1 
1 5  
Applying R1 → R1 + R3 and R2 → R2 − R3 , we To show: P( n) : A = 3n −1 3n −1 3n −1  , n ∈ N
n

have: 2 2
3n −1
3n −1
3 
n −1

 1 0 0  3 −1 1 
0 1 0 =  −15 6 −5 A We shall prove the result by using the principle of
    mathematical induction.
0 0 1  5 −2 2  For n = 1, we have:
 3 −1 1 31−1 31−1 31−1  30 30 30  1 1 1
∴ A −1 =  −15 6 −5    
P(1) : 31−1 31−1 31−1  = 30 30 30  = 1 1 1 = A
 5 −2 2  31−1 31−1 31−1  30 30 30  1 1 1
[5]      
 0 1 n n n–1 Therefore, the result is true for n = 1.
Q. 15. Let A =   , show that (aI + bA) = a I + na
0 0                        Let the result be true for n = k.
bA, where I is the identity matrix of order 2 and n 3k −1 3k −1 3k −1 
∈ N. [NCERT Misc. Ex. Q. 1, Page 100] k  k −1 
P( k ) : A 3 3k −1 3k −1 
Ans. It is given that, 3k −1 3k −1 3k −1 
 
0 1
A=  Now, Ak +1 = A Ak
0 0 
1 1 1 3 3k −1 3k −1 
k −1

To show : P(n) : (aI + bA)n = anI + nan–1 bA, n ∈ N    k −1 k −1 k −1 


= 1 1 1 3 3 3 
We shall prove the result by using the principle of
mathematical induction. 1 1 1 3k −1 3k −1 3k −1 
For n = 1, we have: 3 ⋅ 3k −1 3 ⋅ 3k −1 3 ⋅ 3k −1 
P(1) : (aI + bA) = aI + ba0 A = aI + bA  
= 3 ⋅ 3k −1 3 ⋅ 3k −1 3 ⋅ 3k −1 
Therefore, the result is true.
3 ⋅ 3k −1
3⋅ 3 k −1
3 ⋅ 3 
k −1
For n = 1. 
Let the result be true for n = k., i.e., 3k 3k 3k 
P(k) : (aI + bA)k = akI + kak–1 bA  
= 3k 3k 3k 
Now, we prove that the result is true for n = k + 1. 3 3 3 
k k k
Consider,  
(aI + bA)k + 1 = (aI + bA)k (aI + bA) Therefore, the result is true for n = k + 1.
         = (ak I + kak–1 bA) (aI + bA) Thus by the principle of mathematical induction,
                                = ak + 1I + kakbAI + akbIA + kak–1b2A2 we have:
                                = ak+1I + (k + 1)akbA + kak–1b2A2 …(i) 3n −1 3n −1 3n −1 
0 1 0 1 0 0   
A = 3n −1 3n −1 3n −1  , n ∈ N
Now, A =  = =0
2

0 0  0 0  0 0  3n −1 3n −1 3n −1 
  [5]
From equation (i), we have:
(aI + bA)k + 1 = ak + 1I + (k + 1)akbA + 0  3 − 4   1 + 2 n −4 n 
Q. 17. If A =   Then prove A =  n
n

                                              = ak + 1I + (k + 1) akbA  1 −1  1 − 2 n


Therefore, the result is true for n= k + 1. where n is any positive integer.
Thus, by the principle of mathematical induction,  [NCERT Misc. Ex. Q. 3, Page 100]
we have: Ans. It is given that,
0 1 3 −4 
(aI + bA)n = anI + nan–1bA where A =   ,n∈ N . A=
0 0  
 [5]  1 −1
102 | OSWAAL NCERT SOLUTIONS – Textbook + Exemplar – MATHEMATICS : Class-XII

1 + 2n −4n   2.50 
To prove : P ( n ) : A n =  ,n∈ N
n 1 − 2n  [6000 20000 8000] 1.50 
We shall prove the result by using the principle of 1.00 
mathematical induction. = 6000 × 2.50 + 20000 × 1.50 + 8000 × 1.00
For n = 1, we have:
= 15000 + 30000 + 8000
1 + 2 −4  3 −4 
P (1) : A1 =  = =A = 53000
1 1 − 2  1 −1 
Therefore, the total revenue in market 1 is `46000
Therefore, the result is true for n = 1. and the same in market 2 is `53000. [2½]
Let the result be true for n = k., i.e., (b) The unit cost prices of x, y and z are respectively
1 + 2k −4k  given as `2.00, `1.00 and 50 paise. Consequently,
P ( k ) : Ak =  ,n∈ N
k 1 − 2k  the total cost prices of all the products in market 1
can be represented in the form of a matrix as:
Now, we prove that the result is true for n = k + 1.  2.00 
Consider,
Ak + 1 = Ak A
[10000 2000 18000 ] 1.00 
0.50 
1 + 2k −4k  3 −4 
= = 10000 × 2.00 + 2000 × 1.00 + 18000 × 0.50
k 1 − 2k  1 −1 
= 20000 + 2000 + 9000
3(1 + 2k ) − 4k −4(1 + 2k ) + 4k 
= = 31000
3k + 1 − 2k −4k − 1(1 − 2k ) 
Since the total revenue in market 1 is `46000, the
3 + 6k − 4k −4 − 8k + 4k  gross profit in this market
= 
3k + 1 − 2k −4k − 1 + 2k  = (`46000 – `31000) = `15000.
3 + 2 k −4 − 4k  The total cost prices of all the products in market 2
=
1 + k −1 − 2k  can be represented in the form of a matrix as:
 2.00 
1 + 2( k + 1) −4( k + 1) 
=  [6000 20000 8000] 1.00 
1 + k 1 − 2( k + 1)
0.50 
Therefore, the result is true for n = k + 1.
= 6000 × 2.00 + 20000 × 1.00 + 8000 × 0.50
Thus, by the principle of mathematical induction,
we have: = 12000 + 20000 + 4000
1 + 2n −4n  = Rs 36000
An =  ,n∈N
1 − 2n 
[5] Since the total revenue in market 2 is `53000, the
n gross profit in this market
Q. 18. A manufacturer produces three products x, y and
= (`53000 – `36000) = `17000. [2½]
z which he sells in two markets. Annual sales are
Q. 19. Find the matrix X so that
indicated below:
1 2 3   −7 −8 −9 
Market Products Products Products X = 6 
4 5 6  2 4
.
1 10,000 2,000 18,000
 [NCERT Misc. Ex. Q. 11, Page 101]
2 6,000 20,000 8,000 Ans. It is given that,
(a) If unit sale prices of x, y and z are `2.50, `1.50 and 1 2 3   −7 −8 −9 
`1.00, respectively, find the total revenue in each X = 
market with the help of matrix algebra. 4 5 6  2 4 6
(b) If the unit costs of the above three commodities The matrix given on the RHS of the equation is a
are `2.00, `1.00 and 50 paise respectively. Find the 2 × 3 matrix and the one given on the LHS of the
gross profit. [NCERT Misc. Ex. Q. 10, Page 101] equation is a 2 × 3 matrix. Therefore, X has to be a 2
Ans. (a) The unit sale prices of x, y and z are respectively × 2 matrix.
given as `2.50, `1.50 and `1.00. Consequently, the Now, let
total revenue in market 1 can be represented in the a c 
form of a matrix as: X= 
 2.50 
b d 
[10000 2000 18000 ] 1.50  Therefore, we have:
 a c  1 2 3   −7 −8 −9 
1.00  b d   4 5 6 =  2 4 6
= 10000 × 2.50 + 2000 × 1.50 + 18000 × 1.00     
= 25000 + 3000 + 18000  a + 4c 2a + 5c 3a + 6c   −7 −8 −9 
b + 4d 2b + 5d 3b + 6d  =  2 4 6 
= 46000     [2½]
The total revenue in market 2 can be represented in Equating the corresponding elements of the two
the form of a matrix as: matrices, we have:
MATRICES | 103
a + 4c = −7 , 2a + 5c = −8 3a + 6c = −9 Thus, by the principle of mathematical induction,
we have (AB)n = AnBn, for all natural numbers. [5]
b + 4d = 2, 2b + 5d = 4, 3b + 6d = 6
 0 1 0 −1
            Now, a + 4c = –7 Q. 21. If A =   and B =   , then show that (A
 1 1  1 0
⇒                                          a = –7 – 4c
∴                     2a + 5c = –8 + B) (A – B) ≠ A2 – B2.
⇒ –14 – 8c + 5c = –8  [NCERT Exemp. Ex. 3.3, Q. 9, Page 53]
Ans. We have,
⇒                                   –3c = 6
⇒                                           c = –2 0 1 0 −1
                                     A =   and B =  
∴                                           a = –7 – 4(–2) = –7 + 8 = 1  1 1  1 0
Now,            b + 4d = 2
0 + 0 1 − 1
⇒                                          b = 2 – 4d ( A + B) =  
∴                    2b + 5d = 4  1 + 1 1 + 0
⇒       4 – 8d + 5d = 4 0 0
= 
⇒                                 –3d = 0  2 1 2× 2
                  
⇒                                         d = 0
∴                                           b = 2 – 4(0) = 2 0 − 0 1 + 1
     and ( A − B) =  
Thus, a = 1, b = 2, c = –2 and d = 0  1 − 1 1 − 0
 1 −2  0 2 
Hence, the required matrix X is  . [2½] = 
2 0 0 1 2× 2
Q. 20. If A and B are square matrices of the same order Since, (A + B) (A − B) is defined, if the number
such that AB = BA, then prove by induction that of columns of (A + B) is equal to the number of
ABn = BnA. Further, prove that (AB)n = AnBn for rows of (A − B), so here multiplication of matrices
all n ∈ N. [NCERT Misc. Ex. Q. 12, Page 101] (A + B).(A − B) is possible.
Now,
Ans. A and B are square matrices of the same order such
that AB = BA. 0 + 0 0 + 0 
( A + B ) 2× 2 ⋅ ( A − B ) 2× 2 =  
To prove : P(n) : ABn = BnA, n ∈ N 0 + 0 4 + 1
                                                     For n = 1, we have: 0 0 
                                         P(1) : AB = BA [Given] =  …(i)
0 5 
⇒                                                  AB1 = B1A
Therefore, the result is true for n = 1. A2 = AA
Also,        
Let the result be true for n = k. 0 1 0 1
= ⋅ 
P(k) : ABk = BkA …(i)  1 1  1 1
Now, we prove that the result is true for n = k + 1. 0 + 1 0 + 1
ABk + 1 = ABkB = 
= (BkA)B [By using Eq. (i)] 0 + 1 1 + 1
= Bk(AB) [By associative law] 1 1
= 
= Bk(BA) [AB = BA (Given)] 1 2
                                     
= (BkB)A [By associative law] And
= Bk + 1 A
B2 = B ⋅ B
Therefore, the result is true for n = k + 1.
Thus, by the principle of mathematical induction, 0 −1 0 −1
= ⋅ 
we have  1 0  1 0
ABn = BnA, n ∈ N.
 0 − 1 0 + 0
Now, we prove that = 
(AB)n = AnBn 0 + 0 −1 + 0 
For all n ∈ N  −1 0 
= 
For n = 1, we have : (AB)1 = A1B1 = AB
            0 −1
Therefore, the result is true for n = 1.
Let the result be true for n = k. 1 1  −1 0 
A 2 − B2 =  − 
(AB)k = AkBk …(ii) 1 2   0 −1
Now, we prove that the result is true for n = k + 1.  2 1
(AB)k + 1 = (AB)k(AB) =   …(ii)
         = (AkBk)(AB) [By using Eq. (ii)]  1 3
         = Ak (Bk A)B [By associative law] Thus, we see that
         = Ak (ABk)B [ABn = Bn A for all n ∈ N)] (A + B)(A – B) ≠ A2 – B2 [By using Eqs. (i) and (ii)]
k k 0 0   2 1
         = (A A)(B B) [By associative law]
⇒ ≠ 
= A k + 1 Bk + 1  0 5   1 3
Therefore, the result is true for n = k + 1.
Hence proved. [5]
104 | OSWAAL NCERT SOLUTIONS – Textbook + Exemplar – MATHEMATICS : Class-XII

Q. 22. Find the matrix A satisfying the matrix equation  cos x sin x 
 2 1  −3 2   1 0  P( x ) =  
  A = .  − sin x cos x 
 3 2   5 −3  0 1  cos y sin y 
 [NCERT Exemp. Ex. 3.3, Q. 12, Page 54] ∴ P( y ) =  
 − sin y cos y 
Ans. We have,
 2 1  −3 2   1 0  cos x sin x   cos y sin y 
Now, P( x ) ⋅ P( y ) =   
 3 2  A ⋅  5 −3 = 0 1  − sin x cos x   − sin y cos y 
  2× 2   2× 2   2× 2
 cos x ⋅ cos y − sin x ⋅ sin y cos x ⋅ sin y + sin x ⋅ cos y 
a b  = 
Let A =    − sin x ⋅ cos y − cos x ⋅ sin y − sin x ⋅ sin y + cos x ⋅ cos y 
c d
1  a b   −3 2   1
2 0  cos( x + y ) sin( x + y )
∴ = =  …(i)
3
2   c d   5 −3 0 1  − sin( x + y ) cos( x + y ) 

 2a + c
2b + d   −3 2   1 0 Q cos( x + y ) = cos x ⋅ cos y − sin x ⋅ sin y and 
⇒  +  =   sin( x + y ) = sin x ⋅ cos y + cos x ⋅ sin y 
3b + 2d   5 −3 0
3a 2c 1  
 −6a − 3c + 10b + 5d 4a + 2c − 6b − 3d   1 0  cos( x + y ) sin( x + y )
⇒ =              and P( x + y ) = 
 −9a − 6c + 15b + 10d 6a + 4c − 9b − 6d  0 1   …(ii)
 − sin( x + y ) cos( x + y )
⇒ − 6a − 3c + 10b + 5d = 1  cos y sin y   cos x sin x 
 …(i) Also, P( y ) ⋅ P( x ) =   
⇒                4a + 2c – 6b – 3d = 0 …(ii)  − sin y cos y   − sin x cos x 
⇒ –9a – 6c +15b + 10d = 0 …(iii)
                         6a + 4c – 9b – 6d = 1 …(iv)  cos y ⋅ cos x − sin y ⋅ sin x cos y ⋅ sin x + sin y ⋅ cos x 
=
On adding Eqs. (i) and (iv), we get:  − sin y ⋅ cos x − sin x ⋅ cos y − sin y ⋅ sin x + cos y ⋅ cos x 
                                                    c + b – d = 2
⇒                                                                    d = c + b – 2 …(v)  cos( x + y ) sin( x + y )
=  …(iii)
On adding Eqs. (ii) and (iii), we get:  − sin( x + y ) cos( x + y ) 
                    –5a –4c + 9b + 7d = 0 …(vi) Thus, we see from the Eqs. (i), (ii) and (iii) that,
On adding Eqs. (vi) and (iv), we get:
P(x)·P(y) = P(x + y) = P(y)·P(x)
                                  a + 0 + 0 + d = 1
Hence proved. [5]
⇒                                                                  d = 1 – a …(vii)
From Eqs. (v) and (vii), Q. 24. If possible, using elementary row transformations,
                                                    c + b – 2 = 1 – a find the inverse of the following matrices:
⇒                                         a + b + c = 3 …(viii)  2 −1 3  2 3 −3 
                                                                             a = 3 – b – c (i)  −5 3 1 (ii)  −1 −2 2 
 −3 2 3   1 1 −1
Now, using the values of a and d in Eq. (iii), we get:   
–9(3 – b – c) – 6c + 15b + 10(–2 + b + c) = 0
⇒ –27 + 9b + 9c – 6c + 15b – 20 + 10b + 10c = 0  2 0 −1
⇒ 34b + 13c = 47 …(ix) (iii)  5 1 0 
0 1 3
Now, using the values of a and d in Eqs. (ii), we get:  
      4(3 – b – c)+ 2c – 6b – 3(b + c – 2) = 0  [NCERT Exemp. Ex. 3.3, Q. 51, Page 59]
⇒ 12 – 4b – 4c + 2c – 6b – 3b – 3c + 6 = 0 Ans. For getting the inverse of the given matrix A by
⇒                                                                                       –13b – 5c = –18 …(x) row elementary operations we may write the given
On multiplying Eq. (ix) by 5 and Eq. (x) by 13, then matrix as A = IA
adding, we get:
–169b – 65c = –234  2 −1 3  1 0 0 
170b + 65c = 235 (i) Q  −5 3 1 = 0 1 0  A
b =1  −3 2 3 0 0 1
⇒                 –13 × 1 – 5c = –18 [From Eq. (x)]  2 −1 3  1 0 0
 −3
⇒                                             –5c = –18 + 13 = –5 ⇒  2 4  =  1 1 0  A [Q R2 → R2 + R1 ]
⇒                                                     c = 1
 −3 2 3 0 0 1
∴                                                      a = 3 – 1 –1 = 1 and d = 1 –1 = 0
1 1  2 −1 3  1 0 0
∴A = 
∴                                                      ⇒  −3 2 4  =  1 1 0  A [Q R3 → R3 – R2 ]
1 0  [5]  0 0 −1  −1 −1 1
 cosx sinx 
Q. 23. If P( x ) =   , then show that P(x) P(y)  −1 1 7   2 1 0 
 − sinx cosx   −3 2 4  =  1 1 0  A Q R → R + R
= P(x + y) = P(y)·P(x).
⇒     [ 1 1 2]
 0 0 −1  −1 −1 1
 [NCERT Exemp. Ex. 3.3, Q. 46, Page 58]
Ans. We have,
MATRICES | 105
 −1 1 7  2 1 0  −1   1 
    1 0 2   2 0 0
⇒  0 −1 −17  =  −5 −2 0  A [ R2 → R2 − 3R1 ]      1 
5   −5  R → R1 and 
 0 0 −1  −1 −1 1 ⇒ 0 1 = 1 0 A  1 2
 2  2   
 −1 0 −10   −3 −1 0       R3 → 2R3 
     R → R1 + R2   0 0 1   5 −2 2 
⇒  0 −1 −17  =  −5 −2 0  A  1 
and R3 → −1 ⋅ R3     
 0 0 1  1 1 −1
 −1 0 0   7 9 −10   1 0 0  3 −1 1  1 
     R → R1 + 10 R3   R1 → R1 + 2 R3 and 
⇒  0 −1 0  = 12 15 −17  A  1  ⇒ 0 1 0 =  −15 6 −5 A  
and R2 → R2 + 17 R3  0 0 1  5 −2 2  R → R − 5 R 
 0 0 1  1 1 −1
 
2 2 3
2
 1 0 0   −7 −9 10 
     R1 → −1R1 and  3 −1 1
⇒ 0 1 0  =  −12 −15 17  A  R2 → −1R2 
0 0 1  1 1 −1
 Hence,  −15 6 −5 is the inverse of given
 5 −2 2 
 −7 −9 10 
matrix A. [5]
So, the inverse of A is  −12 −15 17 
  Q. 25. Using elementary row transformations, find the
 1 1 −1
 1 2 3
 2 3 −3  1 0 0  inverse of the matrix A =  2 
5 7 .
(ii) ∴  −1 −2 2  = 0 1 0  A  −2 −4 −5
   
 1 1 −1 0 0 1  [CBSE Board, Delhi Region, 2018]
0 1 −1  1 0 −2  Ans. Given that,
0 −1  R2 → R2 + R3 
⇒  1 = 0 1 1 A  
 1 2 3
 and R → R − 2 R3  
A =  2 5 7
 1 1 −1 0 0 1
1 1

0 1 −1  1 0 −2   −2 −4 −5
0 0 0  =  2 1 −2  A [ R2 → R2 + R1 ]
⇒  We know that, A= IA
 1 1 –1  0 0 1  1 2 3  1 0 0
 2 5 7 0 1 0 
Since, second row of the matrix A on LHS    
containing all zeroes, so we can say that inverse of  −2 −4 −5 0 0 1
matrix of matrix A does not exist.           
 2 0 −1  1 0 0  Apply R2 → R2 – 2R1 and R3 → R3 + 2R1
(iii) ∴  5 1 0  = 0 1 0  A  1 2 3  1 0 0
    0 1 1 = A  −2 1 0 
 0 1 3 0 0 1    
0 0 1  2 0 1
 2 0 −1  1 0 0 
⇒  3 1 1 =  −1 1 0  A [ R2 → R2 − R1 ] Apply R1 → R1 – 2R2
 0 1 3  0 0 1  1 0 1  5 −2 0 
0 1 1 = A  −2
 2 0 −1  1 0 0     1 0 
     R2 → R2 − R1 and 
⇒  1 1 2  =  −2 1 0  A R → R + R  0 0 1  2 0 1
 2 1 2   1 0 1  3 3 1 
Apply R1 → R1 – R3 and R2 → R2 – R3
 2 0 −1  1 0 0   1 0 0  3 −2 −1
  R3 → R3 + R1 and 
5   −5 
   
=
 
⇒ 0 1

= 1 0 A
 R2 → R2 − 1 R1   0 1 0  A  −4 1 −1
2  2  0 0 1  2 0 1
 4 1 1  2 0 1  2 
   
 3 −2 −1
 2 0 −1  1 0 0 
 5   −5  ∴ A −1 =  −4 1 −1
⇒ 0 1 = 1 0 A [ R3 → R3 − 2R1 ]  2 0 1 [6]
 2  2  ∴
 0 1 3  0 0 1
     1 −1 2   −2 0 1
   
    Q. 26. Use product 0 2 −3   9 2 −3  to solve the
 2 0 −1  1 0 0   3 −2 4   6 1 −2 
   
5   −5
⇒ 0 1 = 1 0 A [ R3 → R3 − R2 ] system of equations x + 3z = 9; – x + 2y – 2z = 4;
 2  2 
 1  5  2x – 3y + 4z = –3.
0 0   −1 1
 2  2   [CBSE Board, Delhi Region, 2017]
106 | OSWAAL NCERT SOLUTIONS – Textbook + Exemplar – MATHEMATICS : Class-XII

Ans. and use it to solve the system of equations x – y +


 11 − −11 22   − −22 00 11 z = 4; x – 2y – 2z = 9; 2x + y + 3z = 1.
AB =
AB = 0 0 22 − −33 ⋅⋅  99 22 −
−33  [CBSE Board, All India Region, 2017]
 33 − 2 4   6 1 −2 
 −2 4   6 1 −2   −4 4 4   1 −1 1  8 0 0 
 11 00 00 Ans. Getting  −7 1 3  1 −2 −2  = 0 8 0   …(i)
  5 −3 −1  2 1 3 0 0 8 [1½]
= 00 11 00
=     
0 0 1
0 0 1 Given equations can be written as:
AB =
AB = II [1]  1 −1 1 x   4 
−  1 −2 −2  y  =  9 

⇒ A A −1 =
1
=B 2
B 1 3    
 z   1
−22 00                           [1]
 − 11
=  99 22 −
= −33 ⇒ AX = B
 66 11 − −22 
 [1] From equation (i), we have
Given equations in matrix form are:  −4 4 4 
−1 1 
 1 0 3  x   9  A =  −7 1 3
 −1 2 −2   y  =  4  8 
      5 −3 −1 [1]
 2 −3 4   z   −3 [1] −1
X=A B
AX = C [½]
      X = (A)–1C = (A–1)C [1]  −4 4 4  4 
1  
 x   −2 9 6   9  0  =  −7 1 3  9 
⇒ y =  0 2 1  4  =  5 8  
 z   1 −3 −2   −3  3  5 −3 −1 1 [1]
      
[1]
 24   3 
x = 0, y = 5 and z = 3 [1] 1   
−16  =  −2 
 −4 4 4   1 −1 1 8    
Q. 27. Determine the product  −7 1 3   1 −2 −2   −8   −1 [1]
 5 −3 −1  2 1 3 
  ⇒ x = 3, y = –2 and z = –1 [½]

Some Commonly Made Errors


¾¾ Generally, students confuse in general formula of skew symmetric or symmetric matrices.
¾¾ Students confuse in transpose of a matrix or inverse of a matrix.
¾¾ Students does not use right sign convention of matrix element.
¾¾ Students forget the identity of the matrices.

EXPERT ADVICE
☞ Features of judgment matrix aggregation.
☞ Matrix aggregation method is based on improved GSO Algorithm.
☞ Experimental Analysis of a matrix.
☞ Comparing and combining dimensions of decision matrices.

OSWAAL LEARNING TOOLS


For Suggested Online Videos
Visit : https://goo.gl/tgGuv3 Visit : https://goo.gl/JmxC84

Or Scan the Code Or Scan the Code

  

You might also like